Calculo Avanzado - Capitulo 3

Cap´ıtulo 3 Funciones de varias variables 3.1. Introducci´ on El concepto de funci´on, ya establecido, en cursos prece

Views 65 Downloads 1 File size 608KB

Report DMCA / Copyright

DOWNLOAD FILE

Recommend stories

Citation preview

Cap´ıtulo 3 Funciones de varias variables 3.1.

Introducci´ on

El concepto de funci´on, ya establecido, en cursos precedentes puede ser extendido a situaciones del tipo → − f : A ⊂ IRm → B ⊂ IRn , m, n ∈ IN − en el sentido que son correspondencias que asignan a cada vector → x ∈ → − → − → − → − A un u ´nico vector y ∈ B anotado y = f ( x ) . Consideradas las − − componentes de → x y de → y la igualdad anterior, tambi´en, se anota de la forma → − (y1 , y2 , . . . , yn ) = f (x1 , x2 , . . . , xm ); estas funciones se denominan funciones vectoriales de variable vectorial , o bien, n funciones dependientes de m variables independientes. Sus respectivos dominio de definici´on A y recorrido B se denominan campos vectoriales. En este curso se consider´an funciones del tipo → − f : A ⊂ IRm → B ⊂ IR esto es con m > 1 y n = 1 , que por las caracter´ıstica de los espacios pasan a llamarse, funciones reales de variable vectorial ( de m variables independientes). En la unidad anterior fueron consideradas funciones vectoriales → − f : A ⊂ IR → IRn 189

es decir, con m = 1 y n > 1. Estas funciones se denotan como → − y = f (x1 , x2 , . . . , xm ) − − −x = (x , x , . . . , x ) y se estudiar´an alternativamente → y = f (→ x ) con→ 1 2 m sus propiedades en cu´anto a variaciones, gr´aficas, aplicaciones, etc, a partir de los conceptos de l´ımite, continuidad, y derivada. En particular, para los casos usuales m = 2 y m = 3, se denotan z = f (x, y), (x, y) ∈ A ⊂ IR2 ; w = f (x, y, z)(x, y, z) ∈ A ⊂ IR3 , respectivamente. Ejemplos Caso 1) a) La correspondencia z = f (x, y) = xy , A = IR2 , B = IR , restringidos x e y a valores positivos f (x, y) = xy corresponde al ´area del rect´angulo de lados x e y (comparado con f (r) = πr2 , ´esta representa el a´rea del c´ırculo de radio r y es funci´on del tipo f : A ⊂ IR → B ⊂ IR, o sea con m = 1 y n = 1. b) La correspondencia w = f (x, y, z) = xyz , tiene A = IR3 , B = IR , y con x, y, z positivos representa el volumen V del prisma recto de aristas x, y, z. Caso 2) a) La funci´on f : A ⊂ IR2 → B ⊂ IR dada por p z = f (x, y) = x2 + y 2 , A = IR2 , B = IR,

representa la distancia del punto P = (x, y) al origen O = (0, 0) de IR2 . b) La funci´on f : A ⊂ IR3 → B ⊂ IR dada por p w = f (x, y, z) = x2 + y 2 + z 2 , A = IR3 , B = IR,

representa la distancia del punto P = (x, y, z) al origen O = (0, 0, 0) de IR3 . Caso 3) La funci´on z = f (x, y) = p

1 x2

y2

, en que f : A ⊂ IR2 → B ⊂ IR

+ tiene dominio A = IR2 ,y recorrido B = IR, representa el potencial 190

electrost´atico en cada punto P = (x, y) del plano debido a una carga unitaria colocada en el origen O = (0, 0) de IR2 . Nota: En estos ejemplos de funciones con m = 2, ´o m = 3 se puede considerar los conceptos de l´ıneas de nivel y superficies de nivel como S = {(x, y) ∈ A/ f (x, y) = c} para c constante dada; y S = {(x, y, z) ∈ A/ f (x, y, z) = c} para c constante dada. n

2

p

x2

y2

o

a) En el caso 2) se tiene que S = (x, y) ∈ IR / + = c con √ c ≥ 0 son circunsferencias de radio c (´o el origen si c = 0). ) ( 1 = c, c > 0 b) En el caso 3) S = (x, y) ∈ IR2 − {(0, 0)} / p x2 + y 2 1 las l´ıneas que verifican x2 + y 2 = 2 se llaman l´ıneas equipotenciales c alrededor de la carga, describen circunsferencias centradas el (0, 0) de 1 radio . c Unido a lo anterior, en topograf´ıa las curvas determinadas por ecuaci´on f (x, y) = c se llaman tambi´en contornos p de nivel de una superficie. Por ejemplo, la funci´on z = f (x, y) = 25 − x2 − y 2 que tiene  A = (x, y) ∈ IR2 / x2 + y 2 ≤ 25 , B = [0, 5]

representa na.esf´erica de radio basal 5 y altura 5; con c = 3 puna ”monta˜ se tiene 25 − x2 − y 2 = 3, de lo cual se tiene el contorno de nivel x2 + y 2 = 16.

Caso 4)

La presi´on P ejercida por un gas ideal encerrado en un cilindro pist´on es T donde k cte, T es la temperatura dada por la funci´on P (T, V ) = k V y V el volumen del cilindro. En este caso las l´ıneas de nivel reciben el nombre de l´ıneas isob´aricas P = c, ´o isotermas cuando T = c con T funci´on temperatura en cada punto (x, y, z) . Caso 5) 191

a) El a´rea S de la superficie del cuerpo humano es una funci´on del peso w y la altura h dada por S (w, h) = 0, 091w0,425 h0,725 (deducci´on emp´ırica), donde w est´a en libras y h en pulgadas y S en pie2 . b) Tambi´en S es dada por S (h, t) = 2ht donde h es la altura en cm y t es la longitud de la circunferencia m´axima del muslo en cm. Por ejemplo, una ni˜ na de altura h = 156 cm y 50 cm de de circunsferencia m´axima del muslo tienen una superficie corporal de 15,600 cm2 . Caso 6) La funci´on f (x, y) = C xa y 1−a con C y a constantes, donde x son las unidades de trabajo e y las unidades de capital representa un modelo de unidades de producci´on (Modelo de Cobb-Douglas), que se utiliza en econom´ıa y en la evaluaci´on de proyectos. Caso 7) a) La potencia el´ectrica para un voltage E y resistencia R es la funci´on E2 . de dos variables P (E, R) = R b) La resistencia total al conectar en paralelo dos resistencias R1 , R2 1 1 1 = + , de donde se deduce R = es regido por la ecuaci´on R R1 R2 R (R1 , R2 ) . Caso 8) a) La aceleraci´on centr´ıpeta de una part´ıcula que se mueve en la cirv2 cunferencia de radio r siendo v la rapidez es dada por a (r, v) = . r b) El desplazamiento vertical de una cuerda larga sujeta en el origen, que cae bajo la acci´on de su propio peso es dada por

Caso 9)

  − g (2axt − x2 ) , si 0 ≤ x ≤ at 2a2 g u (x, t) =  − 2 t2 , si x > at, a cte 2a

La concentraci´on molecular C (x, t) de un l´ıquido es dada por x2 t

C (x, t) = t−1/2 e− k

y esta funci´on verifica la ecuaci´on de difusi´on 192

∂C k ∂ 2C = . 4 ∂x2 ∂t Caso 10) Cuando una chimenea de h metros de altura emite humo que contiene contaminante o´xido n´ıtrico la concentraci´on C (x, z) a x km de distancia y z metros de altura es dada por a C (x, z) = 2 x



e

b(z−h)2 x2

+e



b(z−h)2 x2



∂C ∂C Calcular e interpretar los valores de , ,en el punto P = (2, 5) ∂x ∂t para a = 200, b = 0, 02; h = 10 m. Caso 11) Un ejemplo de una funci´on que depende de 4 variables es la que establece la ley de Poiseuille en la cual la intensidad del flujo de un fluido viscoso (como la sangre) trav´es de un conducto (como una arteria), es R4 (p1 − p2 ) Q (R, L, p1 , p2 ) = k L con k cte, R radio de conducto, L su longitud y p1 , p2 presiones en los extremos del conducto.

3.2. Funciones Escalares de Variable Vectorial 3.2.1.

Conceptos Topol´ ogicos

Nuestro espacio universo ser´a Rn pensado preferentemente con n = 2 o n = 3. − − Si → x ∈ Rn entonces cada vector → x = (x , x , . . . , x ) esta conformado 1

2

n

por una n- upla ordenada de n´ umeros reales.

− − La m´etrica que se usar´a, es la m´etrica usual, es decir si → x ,→ y ∈ Rn tal que: → − x = (x1 , x2 , . . . , xn ) → − y = (y1 , y2 , . . . , yn ) 193

entonces − − k→ x −→ yk=

n X i=1

(xi − yi )2

! 21

A kk se le llama norma euclidea y permite calcular la distancia entre los puntos x e y. Si y = 0 entonces − k→ xk=

n X

x2i

i=1

! 21

Vecindad Sea x0 ∈ Rn , una vecindad de x0 es: Vδ (x0 ) = {x ∈ Rn | kx − x0 k < δ} En R2 esta vecindad es un disco centrado en P0 . En R3 es una esfera centrada en P0 y de radio δ Caso especial es el de la vecindad despuntada Vδ∗ (x0 ) = Vδ (x0 ) − {x0 } Punto Interior Sea S ⊆ Rn , x ∈ S es un punto interior de S si existe δ > 0 tal que: Vδ (x) ⊂ S

Conjunto Abierto Sea A ⊆ Rn , diremos que A es un conjunto abierto si y s´olo si todos sus puntos son interiores.

194

Ejemplo: ¿Cu´ ales de los siguientes conjuntos son abiertos? A = {(x, y) ∈ R2 | x2 + y 2 ≤ 1}

No es conjunto abierto

A = {(x, y) ∈ R2 | 0 ≤ x < 1}

No es conjunto abierto

A = {(x, y) ∈ R2 | |x| < 1, |y| < 1} Es conjunto abierto

Conjunto Cerrado Sea B ⊆ Rn , diremos que B es un conjunto cerrado si su complemento Rn − B es abierto. Punto Frontera Sea A ⊆ Rn , x0 es un punto frontera de A si y solo si ∀δ > 0 : Vδ (x0 ) ∩ A 6= ∅ y Vδ (x0 ) ∩ (Rn − A) 6= ∅ Interior de un Conjunto Sea A ⊆ Rn ,el conjunto de todos los puntos interiores de A se llama el o

interior de A y se denota A o sea: o

A = {x | x es punto interior de A}

Frontera de un Conjunto Sea A ⊆ Rn , la frontera de A es el conjunto de todos los puntos frontera de A y se denota F r(A).

Proposici´ on 1 Si S es abierto, no contiene ninguno de sus puntos fronteras. Dem. Directamente de la definici´on de conjunto abierto.

195

Proposici´ on 2 Si S es cerrado, contiene a todos sus puntos fronteras.

Segmento Lineal Si P1 , P2 ∈ Rn , el segmento lineal que une P1 con P2 es P = (1 − t)P1 + P2 , 0 ≤ t ≤ 1. P1 se llama punto inicial y P2 punto terminal.

Linea Poligonal Una poligonal est´a formada por un n´ umero finito de segmentos lineales unidos sucesivamente por sus extremos.

Punto Aislado Sea A ⊆ Rn , x0 es punto aislado de A si y solo si x0 ∈ A y ∃δ > 0 : Vδ∗ (x0 ) ∩ A = ∅

Punto de Acumulaci´ on Sea A ⊆ Rn , x ∈ Rn , x es punto de acumulaci´on de A si toda vecindad despuntada de x contiene puntos de A, es decir: x es punto de acumulaci´on de A ⇐⇒ Vδ∗ (x0 ) ∩ A 6= ∅

Regi´ on Sea R ⊆ Rn ,diremos que el conjunto R es una regi´on si es un conjunto abierto y cualquier par de puntos de ella pueden unirse mediante una l´ınea poligonal, contenida en R.

196

Regi´ on Cerrada Es una regi´on unida con su frontera. Teorema 3.2.1. Sea C ⊆ Rn y C es conjunto cerrado, entonces C contiene en todos sus puntos de acumulaci´on.

3.2.2. Aspectos Geom´ etrico de las Funciones Escalares Sea f : D ⊆ Rn → R esta funci´on a cada x ∈ D, x = (x1 , x2 , . . . , xn ) le asigna una imagen b ∈ R, b un escalar. b = f (x1 , x2 , . . . , xn ) a este tipo de funciones se les llama usualmente campo escalar. Con el objeto de resaltar sus aspectos geom´etricos, especialmente para funciones de dominio en R2 o R3 analizaremos conceptos como, gr´aficas, curvas de nivel, superficies de nivel, trazos, etc. de estas funciones. Sea f : D ⊆ Rn → R si x ∈ D =⇒ x = (x1 , x2 , . . . , xn ), D es el dominio de la funci´on f. f (D) = Im(f ) = {z ∈ R | z = f (x1 , x2 , . . . , xn )}

3.2.3.

Gr´ afica de una Funci´ on

Sea f : D ⊆ Rn → R , definimos la gr´afica de f como el subconjunto de Rn+1 formado por todos los puntos (x1 , x2 , . . . , xn , f (x1 , x2 , . . . , xn )) para cada (x1 , x2 , . . . , xn ) ∈ Rn . Simb´olicamente  Gf = (x1 , x2 , . . . , xn , f (x1 , x2 , . . . , xn )) ∈ Rn+1 | (x1 , x2 , . . . , xn ) ∈ D As´ı, si n = 2 y f es tal que z = f (x, y) entonces su gr´afica ser´a:  Gf = (x, y, z) ∈ R3 | z = f (x, y) 197

3.2.4.

Curvas y Superficies de Nivel

Sea f : D ⊆ Rn → R y C un escalar. Entonces el conjunto de puntos de nivel de valor C ,se define como el conjunto de puntos (x1 , x2 , . . . , xn ) ∈ D para los cuales f (x1 , x2 , . . . , xn ) = C.Simb´olicamente, el conjunto de nivel S = {(x1 , x2 , . . . , xn ) ∈ D | f (x1 , x2 , . . . , xn ) = C} Si n = 2 el conjunto {(x, y) | f (x, y) = C} es una curva de nivel. Si n = 3 el conjunto {(x, y, z) | f (x, y, z) = C} es una superficie de nivel.

Ejemplo: 1.- Trazar la curva de nivel de f (x, y) = x − y, C = 0, 1, 2 · ··

2.- Trazar la curva de nivel de f (x, y) = 10 − x2 − 4y 2 , C = 0, 1, 2 · ·· Es familia de elipses centradas para C < 10, cuyo eje mayor se encuentra sobre el eje x Observaci´ on: Las curvas y las superficies de nivel permiten dar una imagen de la gr´ afica de la funci´ on. 198

3.2.5.

L´ımite

Sea f : D ⊆ Rn → R una funci´on escalar y x0 un punto de acumulaci´on del dominio D. Formularemos la definici´on de l´ımite de una funci´on escalar de la siguiente forma. Definici´ on 3.2.1. La funci´ on f tiene como l´ımite al n´ umero L ∈ R cuando x → x0 , si para cada ǫ > 0, existe δ (ǫ, x0 ) > 0 tal que x ∈ D y 0 < kx − x0 k < δ

entonces

|f (x) − L| < ǫ

Simb´olicamente:

l´ım f (x) = L ⇐⇒ ∀ǫ > 0, ∃δ > 0 :

x→x0

x ∈ D y 0 < kx − x0 k < δ =⇒ |f (x) − L| < ǫ En el caso particular n = 2 esta definici´on es

l´ım

(x,y)→(x0 ,y0 )

f (x, y) = L ⇐⇒ ∀ǫ > 0, ∃δ > 0 : (x, y) ∈ D y 0 < k(x, y) − (x0 , y0 )k < δ =⇒ |f (x, y) − L| < ǫ

Ejemplo Sea f (x, y) = x2 + 2xy. Determinar si existe 199

l´ım

(x,y)→(3,−1)

(x2 + 2xy)

An´ alisis: Si (x, y) → (3, −1) significa que x est´a cercano a 3 e y est´a cercano a −1 por lo tanto el valor de x2 + 2xy debe estar pr´oximo a 3 se espera entonces que si este l´ımite existe debe ocurrir que:  x2 + 2xy = 3 l´ım (x,y)→(3,−1)

Observe que: q q 2 |x − 3| ≤ (x − 3) ≤ (x − 3)2 + (y + 1)2 = k(x, y) − (3, −1)k q q 2 |y + 1| ≤ (y + 1) ≤ (x − 3)2 + (y + 1)2 = k(x, y) − (3, −1)k

por lo cual:

k(x, y) − (3, −1)k < δ =⇒ |x − 3| < δ y |y + 1| < x2 + 2xy − 3 < δ

por otro lado , se tiene 2 x + 2xy − 3 = (x − 3)2 + 2(x − 3)(y + 1) + 4(x − 3) + 6(y + 1) ≤ |x − 3|2 + 2 |x − 3| |y + 1| + 4 |x − 3| + 6 |y + 1|

Sin p´erdida de generalidad se puede poner la condici´on δ < 1 entonces mayorando t´ermino a t´ermino, produce 2 x + 2xy − 3 < |x − 3| + 2 |y + 1| + 4 |x − 3| + 6 |y + 1| < δ + 2δ + 4δ + 6δ = 13δ  ε , Definiendo δ = min 1, 13

Todo lo anterior permite afirmar que: k(x, y) − (3, −1)k < δ ⇒ x2 + 2xy − 3 < 13 x2 + 2xy − 3 ε < 13δ = 13 = ε 13 200

Lo cual prueba que

l´ım

(x,y)→(3,−1)

(x2 + 2xy) = 3

Consideremos otro ejemplo en que usamos tambi´en la definici´on pero otro procedimiento

Ejercicio: Probar que

l´ım

(x,y)→(0,0)



x2 y 2 x2 + y 2



=0

Soluci´ on: Sea ǫ > 0 y

x2

≤ =⇒

Sea δ =



x2 + y 2 , y 2 ≤ x2 + y 2 =⇒ x2 y 2 ≤ x2 + y 2 x2 y 2 ≤ x2 + y 2 , (x, y) 6= (0, 0) x2 + y 2

ε, k(x, y) − (0, 0)k = k(x, y)k =

p

x2 + y 2

p p √ x2 + y 2 < δ =⇒ x2 + y 2 < ε =⇒ x2 + y 2 < ε

Lo que prueba que

2 2 2 2 xy = x y 0 existe δ > 0 tal que ∀x ∈ D : kx − ak < δ =⇒ |f (x) − f (a)| < ε donde x = (x1 , x2 , ..., xn ), a = (a1 , a2 , ..., an ). O lo que es lo mismo: Se tiene que f

es continua en un punto a si:

i) f (a) existe, ii)l´ım f (x) existe y ;

iii)l´ım f (x) = f (a).

x→a

x→a

Es decir, para que una funci´on de varias variables sea continua en un punto debe estar definida all´ı, debe tener l´ımite en ´el y el valor de la funci´on en el punto debe ser igual al valor del l´ımite en ese punto. En R2 podemos enunciar esta propiedad de la siguiente forma. Una funci´on f en continua en un punto interior (x0 , y0 ) de una regi´on R si f (x0 , y0 ) est´a definida y l´ım

(x,y)→(x0 ,y0 )

f (x, y) = f (x0 , y0 )

f ser´a continua en la regi´on R si es continua en cada punto de R. Las funciones que no son continuas se dicen que son discontinuas. Teoremas de Continuidad Son similares a los teoremas para funciones de una variable. Esto significa que, si una funci´on es combinaci´on de otras funciones y estas funciones a su vez son continuas entonces la funci´on es continua excepto en aquellos puntos en los que no est´a definida.

 

x2 y 2 si (x, y) 6= (0, 0) ,estudie Ejemplo. Sea la funci´on f (x, y) = x2 + y 2  0 si (x, y) = (0, 0) la continuidad de f en R2 . Soluci´ on.

205

Claramente si (x, y) 6= (0, 0), f es un cuociente de funciones continuas por lo que tambi´en es continua. Adem´as, la funci´on es continua en (0, 0) pues se demostr´o en 1.4.2, que x2 y 2 = 0 = f (0, 0), (x,y)→(0,0) x2 + y 2 l´ım

Por lo tanto, esta funci´on es continua en todo R2 . Continuando con este mismo ejemplo, podemos usar coordenadas polares para mostrar de que ´este u ´ltimo l´ımite vale cero. Decir que (x, y) → (0, 0) , en coordenadas polares es equivalente a que r → 0 (independientemente del valor de θ). x2 y 2 en coordenadas polares, x2 + y 2 x = r cos θ, y = r sin θ, obtenemos la funci´on Expresando la funci´on f (x, y) =

g(r, θ) =

2 2 r2 cos 2 θ sin 2 θ 2 cos θ sin θ = r cos 2 θ + sin 2 θ cos 2 θ + sin 2 θ

observese que ϕ (θ) =

cos 2 θ sin 2 θ = cos 2 θ sin 2 θ ≤ 1. cos 2 θ + sin 2 θ

est´a acotada, y adem´as ψ (r) = r2

Lo que implica que: l´ımg(r, θ) = l´ımψ(r)ϕ(θ) = 0.

r→0

r→0

Podemos concluir que si ϕ(θ) ≤ M est´a acotada, en una vecindad con centro en el origen, y ψ(r) → 0 cuando r −→ 0,entonces l´ımψ(r)ϕ(∅) = r→0 0.

Continuidad en un conjunto abierto. Diremos una funci´on es continua en un conjunto abierto U ⊆ Rn ,si ella es continua en cada punto del conjunto U . 206

3.2.7.

Derivadas Parciales

Sea f : D ⊆ R2 → R funci´on de dos variables que est´a definida en una vecindad del puntos (x0 , y0 ). La derivada parcial de f respecto de x en (x0 , y0 ) se define por: ∂f f (x0 + h, y0 ) − f (x0 , y0 ) (x0 , y0 ) = l´ım h→0 ∂x h Similarmente, la derivada parcial de f respecto de y en (x0 , y0 ) se define por: f (x0 , y0 + h) − f (x0 , y0 ) ∂f (x0 , y0 ) = l´ım h→0 ∂y h siempre que estos l´ımites existan. Las derivadas de orden superior son una reiteraci´on de la definici´on anterior, es decir, derivadas sucesivas, as´ı:   ∂ ∂f fx (x0 + h, y0 ) − fx (x0 , y0 ) ∂ 2f = l´ ım = h→0 ∂x2 ∂x ∂x h   fy (x0 , y0 + h) − fy (x0 , y0 ) ∂ ∂f ∂ 2f = l´ım = 2 h→0 ∂y ∂y ∂y h   2 ∂ f ∂ ∂f fy (x0 + h, y0 ) − fy (x0 , y0 ) = l´ım = h→0 ∂x∂y ∂x ∂y h Ejemplo: Ejemplo: Calcular las derivadas parciales de: f (x, y) = x4 sin y + cos xy Soluci´ on: De acuerdo con la definici´on debemos calcular (x + h)4 sin y + cos(x + h)y − (x4 sin y + cos xy) h→0 h l´ım

207

Desarrollando el numerador se tiene: x4 sin y + 4x3 h sin y + 6x2 h2 sin y + 4xh3 sin y + h4 sin y + cos(xy) cos(hy) − sin(xy) sin(hy) − x4 sin y − cos(xy) = 4x3 h sin y + 6x2 h2 sin y + 4xh3 sin y + h4 sin y + cos(xy)(cos(hy) − 1) − sin(xy) sin(hy) = Calculando l´ımite (x + h)4 sin y + cos(x + h)y − (x4 sin y + cos xy) l´ım h→0 h sin(hy) (cos(hy) − 1) − sin(xy)l´ım = 4x3 sin y + cos xyl´ım h→0 h→0 h h Como l´ım

h→0

(cos(hy) − 1) =0 y h

l´ım

h→0

sin(hy) =y h

Tenemos que este l´ımite y por tanto la derivada es ∂f (x, y) = 4x3 sin y − y sin(xy) ∂x De igual forma se puede calcular por definici´on. ∂f (x, y) = x4 cos y − x cos xy ∂y Derivadas Parciales Cruzadas Teorema 3.2.6. (Teorema de Schwarz) Sea f : D ⊆ R2 → R una funci´ on, D abierto. Si las derivadas ∂ 2f existen y son continuas en D, entonces: ∂y∂x ∂ 2f ∂ 2f = ∂x∂y ∂y∂x 208

∂ 2f y ∂x∂y

Demostraci´on queda propuesta. Como ejemplo ilustrativo considere la funci´on

f (x, y) =

(

x3 y−xy 3 x2 +y 2

0

si (x, y) 6= (0, 0) si (x, y) = (0, 0)

y verifique que: ∂ 2f (0, 0) = 1, ∂x∂y

∂ 2f (0, 0) = −1 ∂y∂x

Observaci´ on: Use la definici´on y encuentre estos resultados.

3.3.

Diferenciabilidad en dos variables

Sea f una funci´on definida en una vecindad de (x0 , y0 ). Diremos que f es diferenciable en (x0 , y0 ) si existen n´ umeros A y B tales que: f (x0 + h, y0 + k) − f (x0 , y0 ) = Ah + Bk + α (h, k) k(h, k)k y si el residuo tiene la propiedad

l´ım α (h, k) = 0. Diremos que f

k(h,k)k→0

es diferenciable en una regi´on, si es diferenciable en cada punto de la regi´on. Teorema 3.3.1. Si f es diferenciable en (x0 , y0 ), entonces f es continua en (x0 , y0 ). Demostraci´ on: Sea (x, y) ∈ Vδ ((x0 , y0 )) y f (x0 +h, y0 +k)−f (x0 , y0 ) = A (x − x0 )+B (y − y0 )+α (h, k) k(x, y) − (x0 , y0 )k Adem´as sabemos que |x − x0 | ≤ k(x, y) − (x0 , y0 )k , |y − y0 | ≤ k(x, y) − (x0 , y0 )k entonces 209

kf (x0 + h, y0 + k) − f (x0 , y0 )k ≤ |Ah + Bk + α (h, k)| k(x, y) − (x0 , y0 )k Si (x, y) → (x0 , y0 ) , tenemos que α (h, k) → 0 y k(x, y) − (x0 , y0 )k → 0 por lo tanto kf (x0 + h, y0 + k) − f (x0 , y0 )k → 0 l´ım



(h,k)→(0,0)

f (x0 + h, y0 + k) = f (x0 , y0 )

La diferenciabilidad es tambi´en una condici´on m´as fuerte que la existencia de las derivadas parciales. Teorema 3.3.2. Si f es diferenciable en (x0 , y0 ), entonces las derivadas parciales de primer orden existen en (x0 , y0 ) y fx (x0 , y0 ) = A fy (x0 , y0 ) = B Demostraci´ on Sea f diferenciable en (x0 , y0 ) =⇒ f (x0 + h, y0 + k) − f (x0 , y0 ) = Ah + Bk + α (h, k)



h2 + k 2

dividiendo por h 6= 0 y haciendo k = 0 se tiene: f (x0 + h, y0 ) − f (x0 , y0 ) |h| =A+α h h |h| f (x0 + h, y0 ) − f (x0 , y0 ) =⇒ − A = |α (h)| = |α (h)| h h

pero l´ım α (h) = 0 h→0

f (x0 + h, y0 ) − f (x0 , y0 ) =A h→0 h

∴ l´ım De manera analoga

210

f (x0 , y0 + k) − f (x0 , y0 ) =B k→0 k

∴ l´ım

Se puede tener tambi´en el siguiente criterio para la diferenciabilidad de una funci´on. Teorema 3.3.3. Sea f : D ⊆ R2 → R una funci´ on. Si f tiene primeras derivadas parciales continuas en una regi´on D. Entonces f es diferenciable en cada punto de D. Nota: Todas las ideas dadas para funciones de dos variables se pueden extender a funciones definidas en un espacio n dimensional. Diferencial Total En dos dimensiones: Si f es diferenciable en (x0 , y0 ) entonces la diferencial total es: df =

∂f ∂f dx + dy en (x0 , y0 ) ∂x ∂y

En tres dimensiones:. Si f es diferenciable en (x0 , y0 , z0 ) entonces la diferencial total es: df =

3.3.1.

∂f ∂f ∂f dx + dy + dz en (x0 , y0 , z0 ) ∂x ∂y ∂z

Derivada Direccional

Gradiente Sea f : D ⊆ Rn → R, P ∈ D abierto y f diferenciable en P. Entonces el vector gradiente de f en P se denota ∇f (P ) y se define por la f´ormula: ∇f (P ) =



∂f ∂f ∂f (P ) , (P ) , ..., (P ) ∂x1 ∂x2 ∂xn 211



Ejemplo: Sea f (x, y, z) = 3x3 y 2 z. Calcular ∇f (1, 2, 3). Soluci´ on:

La funci´on tiene derivadas parciales continuas, entonces ∇f (x, y, z) = 9x2 y 2 z, 6x3 yz, 3x3 y 2



Evaluando, el gradiente de la funci´on f en el punto (1, 2, 3) queda ∇f (1, 2, 3) = (108, 36, 12) = 12 (9, 3, 1) Propiedades Si f y g son funciones diferenciables demuestre que: ∇ (f g) = f ∇g + g∇f   f g∇f − f ∇g ∇ = si g no es cero g g2 Ejemplo Si f es diferenciable en una variable y g = f (x2 + y 2 + z 2 ) . Calcular ∇g · ∇g.

Soluci´ on. Si ponemos u = x2 + y 2 + z 2 al derivar parcialmente se tiene ∂g ∂g ∂g = 2xf ′ (u) ; = 2yf ′ (u) ; = 2zf ′ (u). ∂x ∂y ∂z =⇒ ∇g = 2f ′ (u)(x, y, z) =⇒ ∇g · ∇g = 4 [f ′ (u)] 2 [(x, y, z) · (x, y, z)] = 4 [f´(u)] 2 (x2 + y 2 + z 2 ) (Definici´on Derivada direccional) Sea f : D ⊆ Rn → R y P0 un punto interior de D. Sea u b vector unitario kb uk = 1. La derivada direccional de f en P0 en la direcci´on de u b se define por: f (P0 + hb u) − f (P0 ) h→0 h Si este l´ımite existe f tiene derivada direccional en P0 en la direcci´on ∂f (P0 ) . de u b y la denotamos ∂b u l´ım

212

∂f Teorema 3.3.4. Si f es diferenciable en P, entonces (P ) existe ∂b u para todos los vectores u b, unitarios y ∂f (P ) = ∇f (P ) · u b ∂b u

Demostraci´ on:

Considerando la demostraci´on en R3 .Sean P0 = (x, y, z) y u b = (u1, u2 , u3 )

Por definici´on de diferenciabilidad se tiene:

f (P0 + hb u) − f (P0 ) = f (x + hu1 , y + hu2 , z + hu3 ) − f (x, y, z) ∂f ∂f ∂f h · u1 + h · u2 + h · u3 + α |h| = ∂x ∂y ∂z   ∂f ∂f ∂f = h u1 + u1 + u3 + α |h| ∂x ∂y ∂z Dividiendo por h y tomando l´ımite cuando h → 0 se tiene α → 0 y ∂f ∂f ∂f f (P0 + hb u) − f (P0 ) = u1 + u2 + u3 h→0 h ∂x ∂y ∂z l´ım



∂f (P0 ) = ∇f (P0 ) · u b ∂b u

∂f Nota: Para cada u b vector unitario fijo, la derivada direccional de∂b u fine una nueva funci´on, a la cual a su vez se le puede aplicar la definici´on de derivada direccional y tenemos as´ı las derivadas direccionales de orden superior. ∂ ∂ 2f = 2 ∂b u ∂b u



∂f ∂b u



Ejemplo Sea f (x, y) = x3 y 2 , calcular la derivada direccional de f en el punto − P0 (−1, 2) en la direcci´on del vector → u = (4, −3). Soluci´ on.

Como f (x, y) = x3 y 2 en una funci´on diferenciable ∀ (x, y) ∈ R2 ,entonces 213

∂f (P ) = ∇f (P ) · u b ∂b u

En primer lugar calculemos el vector gradiente ∇f (P0 ) = (3x2 y 2 , 2x3 y)

En segundo lugar calculemos el vector unitario → − u (4, −3) u b= → = − 5 kuk Se deduce que

 (4, −3) ∂f (x, y) = 3x2 y 2 , 2x3 y · ∂b u 5

Por lo tanto, evaluando en P0 (−1, 2) queda

 (4, −3) 3 (−1)2 22 , 2 (−1)3 2 · 5 96 12 108 = + = 5 5 5 Teorema 3.3.5. Supongamos que f tiene segundas derivadas parciales continuas en una vecindad de un punto P. Entonces: ∂f (−1, 2) = ∂b u

∂ 2f (P ) = (b u · ∇)2 f 2 ∂b u Demostraci´ on: ∂f = f 1 u1 + f 2 u2 + f 3 u3 ∂b u ∂f1 ∂f2 ∂f3 ∂ 2f = u1 + u2 + u3 2 ∂b u ∂b u ∂b u ∂b u 2 ∂ f = u1 (f11 u1 + f12 u2 + f13 u3 ) + u2 (f21 u1 + f22 u2 + f23 u3 ) ∂b u2 +u3 (f31 u1 + f32 u2 + f33 u3 ) Como las derivadas cruzadas son iguales se tiene ∂ 2f = u21 f11 + 2u1 u2 f12 + 2u1 u3 f13 + u22 f22 + 2u2 u3 f23 + u23 f33 ∂b u2  2 ∂ ∂ ∂ = u1 + u2 + u3 f = (b u · ∇)2 f ∂x ∂y ∂z 214

∂f (P ) = ∇f (P ) · u b ∂b u ∂f 1.) Si ∇f (P ) = 0 =⇒ → (P ) = 0 ∀b u ∂− u 2.) La direcci´on de m´aximo crecimiento de la derivada direccional viene dada ∂f (P ) = k∇f (P )k en la direcci´on del vector gradiente. por ∂b u 3.) La direcci´on de mayor decrecimiento (o m´ınimo crecimiento) viene dada en ∂f la direcci´on −∇f (P ) y el valor m´ınimo es (P ) = − k∇f (P )k . ∂b u Consecuencias de

Demostraci´ on: Basta considerar

3.3.2.

∂f (P ) = ∇f (P ) · u b = k∇f (P )k cos θ ∂b u

Plano tangente y recta normal

Sea z = f (x, y) la ecuaci´on de una superficie cualquiera. En lo que sigue conviene para mayor comprensi´on del razonamiento usar la expresi´on F (x, y, z) = 0 para denotar la superficie de nivel de la funci´on F (x, y, z) = f (x, y) − z = 0 o´ F (x, y, z) = z − f (x, y) = 0 Sea entonces la superficie F (x, y, z) = 0 y P0 = (x0 , y0 , z0 ) un punto − de ella. Sea → r (t) = (x(t), y(t), z(t)) la ecuaci´on param´etrica de una curva C en dicha superficie y que pasa por el punto (x0 , y0 , z0 ). Tomando la diferencial de F (x, y, z) = 0 tenemos ∂F ∂F ∂F dx + dy + dz = 0 ∂x ∂y ∂z − En P0 = (x0 , y0 , z0 ) de la curva donde → r (t0 ) = P0 se tiene 215

∂F (P0 ) ∂F (P0 ) ∂F (P0 ) · x´(t0 )dt + · y´(t0 )dt + · z´(t0 )dt = 0 =⇒ ∂x ∂y ∂z ∂F (P0 ) ∂F (P0 ) ∂F (P0 ) · x´(t0 ) + · y´(t0 ) + · z´(t0 ) = 0 ∂x ∂y ∂z Es decir 

 ∂F (P0 ) ∂F (P0 ) ∂F (P0 ) · (x´(t0 ), y´(t0 ), z´(t0 )) = 0 , , ∂x ∂y ∂z − ∴ ▽F (P ) · → r´(t ) = 0 0

0

− Esto nos dice que ▽F (P0 ) es perpendicular a → r´(t0 ). Esto ocurre para − toda curva C que pase por P0 y como → r´(t0 ) es vector tangente a C se tiene que ▽F (P0 ) es perpendicular a toda recta tangente a la superficie en P0 . Por lo tanto, ▽F (P0 ) es un vector normal a la superficie en P0 .

Plano Tangente La expresi´on de la forma: ∂F (P0 ) ∂F (P0 ) ∂F (P0 ) (x − x0 ) + (y − y0 ) + (z − z0 ) = 0 ∂x ∂y ∂z es la ecuaci´on del plano tangente a la superficie F (x, y, z) = 0 en el punto P0 = (x0 , y0 , z0 ). Si la superficie es z = f (x, y) =⇒ F (x, y, z) = z − f (x, y) = 0 la ecuaci´on del plano tangente se puede escribir, tambi´en como: z − z0 = fx (x0 , y0 )(x − x0 ) + fy (x0 , y0 )(y − y0 ) Recta Normal Sean P0 (x0 , y0 , z0 ) un punto dado y el vector director de esta recta ▽F (P0 ) La ecuaci´on vectorial es:

216

(x, y, z) = (x0 , y0 , z0 ) + t(

∂F (P0 ) ∂F (P0 ) ∂F (P0 ) , , ), ∂x ∂y ∂z

t∈R

Ecuaci´on param´etrica:

∂F (P0 ) ∂x ∂F (P0 ) , y = y0 + t ∂y ∂F (P0 ) x = z0 + t ∂z x = x0 + t

t∈R

Eliminando el par´ametro t obtenemos la ecuaci´on cartesiana: x − x0 y − y0 z − z0 = = Fx (P0 ) Fy (P0 ) Fz (P0 )

Ejemplo: Sea z = ex (cos y + 1)

en el punto (0, π2 , 1) ,calcular las

ecuaciones del plano tangente y la recta normal a la superficie. Soluci´ on. La componentes del vector gradiente son

fx (x, y) = ex (cos y + 1), fy (x, y) = −ex (seny) π π fx (0, ) = 1, fy (0, ) = − 1 2 2 Reemplanzando t´erminos en la ecuaci´on del plano tenemos: z−1

π 1 · (x − 0) + (−1)(y − ) 2 π =⇒ z − 1 = x − y + 2 =

Finalmente, la ecuaci´on del plano tangente queda 217

x−y−z+ y

π +1=0 2

π (x, y, z) = (0, , 1) + t(1, −1, −1) 2

la ecuaci´on de la recta normal en su forma vectorial.

3.3.3.

Funci´ on Compuesta. La Regla de la Cadena.

En este m´odulo abordaremos en forma b´asica la siguiente problem´atica. Si u es una funci´on diferenciable de variables x, y, z, · · ·, y a su vez estas u ´ltimas son funciones de otras variables nuevas t y/o s ¿podemos encontrar la primera derivada parcial de u con respecto a las nuevas variables t y/o s expresada en t´erminos de las derivadas parciales de las funciones dadas?. As´ı, por ejemplo, si un fen´omeno f´ısico est´a ocurriendo digamos en una regi´on cil´ındrica, resultar´ıa mejor expresar las cantidades que interesen en t´erminos de coordenadas cilindricas y no en cartesianas.

Caso particular: Sea f : D ⊆ R2 −→ R tal que u = f (x, y), tiene dos variables independientes. Supongamos que cada una de estas variables es diferenciable de una simple variable independiente t. Si x = x(t) y y = y(t), la derivada de la funci´on compuesta (regla de la cadena) con respecto a t es

∂u dx ∂u dy du = · + · dt ∂x dt ∂y dt du − = ∇u · → r ′ (t) dt

218

Caso particular de dos variable independientes simples. Sea f : D ⊆ R2 −→ R tal que u = f (x, y), tiene dos variables independientes. Estas variables a su vez son funciones diferenciables de dos variables simples independientes t y s. La expresi´on de las derivadas de la funci´on compuesta (o regla de la cadena) es: ∂u dx ∂u dy ∂u = · + · ∂t ∂x dt ∂y dt ∂u ∂u dx ∂u dy = · + · ∂s ∂x ds ∂y ds Las expresiones anteriores las podemos expresar matricialmente como: 

∂u ∂u , ∂t ∂s

Ejemplo 1: Hallar



=



∂u ∂u , ∂x ∂y





dx  dt  dx ds

dz x , si z = , donde x = et dt y

 dy dt  dy  ds y y = ln t.

Soluci´ on. Aplicando la derivaci´on compuesta, tenemos   dz 1 t x 1 = e + − 2 dt y y t   1 et et 1 1 dz = − = et − 2 dt ln t (ln t) t ln t t(ln t)2

dz ∂u dx ∂u dy = · + · =⇒ dt ∂x dt ∂y dt

Ejemplo 2: Hallar y = ets

∂u y ∂t

∂u ∂s

Soluci´ on. 219

si u = f (x, y), donde x = t2 −s2 ,

En este caso dx dy ∂u ∂u = fx (x, y) +fy (x, y) =⇒ = fx (x, y)(2t)+fy (x, y)(s ets ) ∂t dt dt ∂t Similarmente ∂u dx dy ∂u = fx (x, y) +fy (x, y) =⇒ = fx (x, y)(−2s)+fy (x, y)(t ets ) ∂s ds ds ∂s

De forma m´ as general si f : D ⊆ Rn −→ R tal que z = f (y1, y2, y3, · ··, yn, ), es funci´on de n variables independientes. Supongamos tambi´en que cada una de estas variables independientes es funci´on diferenciable de otras m variables simples independientes x1 , x2 , x3 , · · ·, xm . La expresi´on de la derivada de la funci´on compuesta (regla de la cadena) es similar, pudiendose escribir cada una de estas derivadas por n

X ∂z ∂yi ∂z = · , j = 1, 2, ..., m ∂xj ∂y ∂x i j i=1 Esta ecuaci´on se puede escribir utilizando matrices, como



∂z ∂z ,..., ∂x1 ∂xm



=



∂z ∂z ,..., ∂y1 ∂yn



    

∂y1 ... ∂x1 .. . ∂yn ··· ∂x1

∂y1 ∂xm .. . ∂yn ∂xm

     

 ∂yi se denomina matriz jacobiana La matriz n × m ∂xj i=1...n, j=1...m de la transformaci´on yi = yi (x1, x2, x3, · ··, xm ). 

220

Ejemplo 3: Sea f (x, y, z) = x + x2 y + zey , en donde x, y, y z , est´an relacionadas con u y v a trav´es de la transformaci´on x = uv, y = u2 − v 2 , Calcule

∂f ∂f , , y ∂u ∂v

z = u sin v

∂ 2f , en el punto (u, v) = (2, 2). ∂v∂u

Soluci´ on. La matriz jacobiana para esta transformaci´on es     v u xu xv  yu yv  =  2u −2v  senv u cos v zu zv     2 2 xu xv  yu yv  (2, 2) =  4 −4  sin 2 2 cos 2 zu zv Para (u, v) = (2, 2), los valores de x, y y z son 4, 0 , y 2sen2 respectivamente.Entonces la matriz regl´on (fx fy fz ) es  (fx fy fz ) = 1 + 2xy x2 + zey ey

evaluando en el punto (4, 0, 2sen2) queda  1 + 2xy x2 + zey ey (4, 0, 2sen2) = (1

16 + 2sen2

1)

De donde (fu



 xu xv fv ) = (fx fy fz )  yu yv  zu zv = (fx xu + fy yu + fz zu f x xv + f y yv + f z z v )

Evaluando (fu

fv ) (2, 2) = (66 + 9 sin 2

− 62 − 8 sin 2 + 2 cos 2)

Por lo tanto ∂f (2, 2) = 66 + 9 sin 2 ∂u

y

∂f (2, 2) = −62 − 8 sin 2 + 2 cos 2 ∂v 221

Ahora bien, como fu

= f x xu + f y yu + f z z u = (1 + 2xy )v + (x2 + zey )2u + ey sin v

Se tiene ∂ 2f = fuv = 2(xyv + yxv )v + 1 + 2xy + 2(2xxv + zey yv + ey zv )u ∂v∂u + yv ey senv + ey cos v Evaluando en el punto (2, 2) se tiene ∂ 2f (2, 2) = 1 − 36sen2 + 9 cos 2 ∂v∂u

3.3.4.

Funci´ on Impl´ıcita

Cuando definimos funciones en forma impl´ıcita decimos, por ejemplo, sea y = ϕ(x) una funci´on diferenciable definida impl´ıcitamente por medio de la ecuaci´on F (x, y) = 0, o bien, sea z = f (x, y), definida impl´ıcitamente por la ecuaci´on F (x, y, z) = 0. El primer caso lo podemos ejemplificar con una funci´on y = ϕ(x) definida por la ecuaci´on de la hip´erbola x2 − 4xy − 3y 2 = 9.

Muchas veces las ecuaciones que relacionan a las variables pueden ser tan complejas y no lineales, que no es posible esperar encontrar relaciones sencillas, si las hay, expl´ıcitas que expresen a una variable en t´erminos de las otras. En t´erminos generales, lo que se pide es que esta relaci´on entre las variables exista localmente, es decir en alguna vecindad de un punto donde las ecuaciones se satisfacen. La mayor´ıa de las veces no se esperan resultados globales, es decir, no todos los puntos que satisfacen las relaciones impl´ıcitas satisfacen tambi´en las expl´ıcitas. Con el objeto de ilustrar estas ideas analicemos el siguiente ejemplo.

Ejemplo: Pruebe que la ecuaci´on z 5 + z + xy = 0 define una funci´on z = f (x, y) para todos los valores x e y. Soluci´ on. 222

Se debe establecer que para cada x e y se puede resolver en forma u ´nica para z la ecuaci´on de quinto grado. Ya que esto no es obvio, razonamos utilizando la gr´afica de u = z5 + z + a gr´afica pendiente Como u´(z) = 5z 4 + 1, ∀a =⇒ u es estrictamente creciente, entonces para cada a existen z1 y z2 tal que u(z1 ) < 0 < u(z2 ) por lo cual existe un u ´nico z tal que u(z) = 0 para cada a . Si se identifica a con la cantidad xy se ha establecido el hecho planteado. Luego, ”z = f (x, y) se define impl´ıcitamente por la ecuaci´on z 5 + z + xy = 0”

Derivaci´ on impl´ıcita Sea F : U ⊆ R2 → R definida en un conjunto abierto U . Sea P0 = (x0 , y0 ) ∈ U un punto tal que: i) F (x0 , y0 ) = 0 ∂F ∂F ii) , son continuas en alguna vecindad Vδ (P0 ) y ∂x ∂y ∂F iii) (x0 , y0 ) 6= 0. ∂z Entonces existe una vecindad (x0 − δ, x0 + δ) de x0 , (y0 − a, y0 + a) vecindad de y0 , y una funci´on u ´nica f (impl´ıcita) de clase C 1 tal que: a) f (x0 ) = y0

y f (x) ∈ (y0 − a, y0 + a)

b) F (x, f (x)) = 0

∀x ∈ (x0 − δ, x0 + δ)

∀x ∈ (x0 − δ, x0 + δ)

c) ∀x, ∈ Vδ (x0 ) tiene derivadas que pueden calcularse como: 223

dy Fx (x, y) =− dx Fy (x, y) En el caso de funciones de dos variables definida impl´ıcitamente, el siguiente teorema afirma la existencia de funciones impl´ıcitas bajo ciertas circunstancias y da f´ormulas para obtener las derivadas parciales de estas funciones. Teorema 3.3.6. (Teorema de la funci´ o impl´ıcita) 3 Sea F : U ⊆ R → R definida en un conjunto abierto U . Sea P0 = (x0 , y0 , z0 ) ∈ U un punto tal que: i)

F (x0 , y0 , z0 ) = 0

∂F ∂F ∂F , , son continuas en alguna vecindad Vδ (P0 ) y ∂x ∂y ∂z ∂F iii) (x0 , y0 , z0 ) 6= 0. ∂z Entonces existe una vecindad V = Vδ (x0 , y0 ), y una vecindad (z0 − a, z0 + a) de z0 y una funci´ on u ´nica f (impl´ıcita) de clase C 1 sobre V tal que: ii)

a) f (x0 , y0 ) = z0

y f (x, y) ∈ (z0 − a, z0 + a)

b) F (x, y, f (x, y)) = 0,

∀(x, y) ∈ V.

∀(x, y) ∈ V.

c) ∀(x, y) ∈ V = Vδ (x0 , y0 ) tiene derivadas que pueden calcularse como: ∂F (x, y, f (x, y)) ∂f (x, y) = − ∂x ∂F ∂x (x, y, f (x, y)) ∂z ∂F (x, y, f (x, y)) ∂f (x, y) ∂y =− ∂F ∂y (x, y, f (x, y)) ∂z Este teorema se puede generalizar a m´as variables, de tal modo que si se dan las condiciones exigidas por el teorema y u = f (x, y, z, · · ·) 224

est´a definida impl´ıcitamente por F (x, y, z , · · ·, u) = 0 y Fu (x, y, z , · · ·, u) 6= 0, entonces: ∂F ∂F ∂f ∂f ∂y = − ∂x , =− ∂F ∂y ∂F ∂x ∂z ∂z ,etc.

Ejemplo. Calcular ecuaci´on

∂z y ∂x

∂z si z se define impl´ıcitamente en la ∂y

x2 y − 8xyz = yz + z 3 Soluci´ on. Tenemos que: F (x, y, z) = −x2 y + 8xyz + yz + z 3 Derivando parcialmente con respecto a x , y ,z.

Fx (x, y, z) = 8yz − 2xy, Fy (x, y, z) = 8xz + z − x2 , Fz (x, y, z) = 8xy + y + 3z 2 Claramente estas derivadas son continuas en R3 , son funciones polin´omicas, Por consiguiente, F pertenece a C1 . Para todo (x, y, z) en que Fz (x, y, z) 6= 0 se tiene. ∂z 8yz − 2xy =− ∂x 8xy + y + 3z 2 ∂z 8xz + z − x2 =− ∂y 8xy + y + 3z 2 225

3.3.5.

Jacobiano

Si f1 , f2 , f3, . . . , fn son funciones diferenciables de Rn en R y si (x1 , x2 , x3 , . . . , xn ) ∈ D su dominio tal que f1 = f1 (x1 , x2 , x3 . . . , xn ) f2 = f2 (x1 , x2 , x3 . . . , xn ) .. . fn = fn (x1 , x2 , x3 . . . , xn ) El Jacobiano de las funciones f1 , f2 , f3, . . . , fn , respecto de las variables x1 , x2 , x3 . . . , xn se define por el determinante de las primeras ∂(f1 , f2 , f3, . . . , fn ) derivadas parciales y se denota , ∂(x1 , x2 , x3 . . . , xn ) ∂f1 ∂f1 ∂f1 . . . ∂x ∂x2 ∂x n 1 ∂f2 ∂f2 ∂f2 ∂(f1 , f2 , f3, . . . , fn ) ∂x1 ∂x2 . . . ∂xn = .. ∂(x1 , x2 , x3 . . . , xn ) ∂f ∂f . ∂fn n n . . . ∂x1 ∂x2 ∂xn Dos funciones definidas impl´ıcitamente Teorema 3.3.7. Sean F , G: U ⊆ R4 → R definidas en un conjunto abierto F (x, y, u, v) y G(x, y, u, v). Sea P0 = (x0 , y0 , u0 , v0 ) ∈ U un punto tal que: i)

F (x0 , y0 , u0 , v0 ) = 0

y G(x0 , y0 , u0 , v0 ) = 0

ii) F y G tienen derivadas parciales continuas en alguna vecindad Vδ (P0 ) y ∂ (F, G) (x0 , y0 , u0 , v0 ) 6= 0. iii) ∂ (u, v) Entonces las ecuaciones F (x, y, u, v) = 0 , G(x, y, u, v) = 0 definen funciones impl´ıcitas u = u(x, y), v = v(x, y) en alguna vecindad Vδ (x0 , y0 ), las cuales tienen primeras derivadas parciales continuas con respecto a cada una de las variables, tal que : a) u0 = u(x0 , y0 ), v0 = v(x0 , y0 ). b) F (x, y, u (x, y) , v (x, y)) = 0 , G(x, y, u (x, y) , v (x, y)) = 0, Vδ (x0 , y0 ) 226

c) Las derivadas parciales de u y v en Vδ (P0 ) est´ an dadas por ∂(F, G) ∂u ∂(x, v) , = − ∂(F, G) ∂x ∂(u, v) ∂(F, G) ∂v ∂(u, x) , = − ∂(F, G) ∂x ∂(u, v)

∂u ∂y

∂v ∂y

∂(F, G) ∂(y, v) =− ∂(F, G) ∂(u, v) ∂(F, G) ∂(u, y) =− ∂(F, G) ∂(u, v)

∂v ∂u y donde u y v Ejemplo Calcule las derivadas parciales ∂x ∂x est´an definidas por las ecuaciones impl´ıcitas x2 + 2uv = 1, x3 − u3 + v 3 = 1 Soluci´ on. En este caso F (x, u, v) = x2 + 2uv − 1 = 0 y G(x, u, v) = x3 − u3 + v3 − 1 = 0 ∂(F, G) 2v 2u = = 6(u3 + v 3 ) −3u2 3v 2 ∂(u, v)

∂(F, G) 6= 0 para todo (x, u, v) donde u 6= 0 y v 6= 0. ∂(u, v) ∂(F, G) 2x 2u = 2 = 6x(v 2 − xu) 3x 3v 2 ∂(x, v) ∂(F, G) 2v 2x 2 = 2 2 = 6x(xv + u ) −3u 3x ∂(u, x)

Entonces, derivando u y v con respecto a x, obtenemos ∂u 6x(v 2 − xu) x(xu − v 2 ) = =− ∂x 6(u3 + v 3 ) u3 + v 3 y 227

6x(xv + u2 ) x(xv + u2 ) ∂v =− = − ∂x 6(u3 + v 3 ) u3 + v 3

3.3.6.

M´ aximos y M´ınimos

M´aximos y M´ınimos (extremos locales o relativos) para funciones de dos o m´as variables. Sea f : U ⊆ R2 → R, U conjunto abierto.

i) f tiene un m´aximo local en x0 ∈ U si f (x0 ) ≥ f (x) ∀x ∈ Bδ (x0 , y0 ) ii) f tiene un m´ınimo local en x0 ∈ U si f (x0 ) ≤ f (x) ∀x ∈ Bδ (x0 , y0 )

Ejemplo 1: La funci´on f (x, y) = 4 − x2 − y 2 tiene un m´aximo local en (0, 0), pues ∆f = f (0,0) − f (x, y) = x2 + y 2 ≥ 0 ya que x2 ≥ 0, y 2 ≥ 0 ∀(x, y) ∈ Bδ (0, 0). Ejemplo 2: La funci´on z = f (x, y) = x2 + y 2 Tiene un m´ınimo local en (0, 0) : f (0, 0) ≤ x2 + y 2 = f (x, y)

f (0, 0) = 0 es m´ınimo absoluto.

Ejemplo 3: La funci´on z = f (x, y) = 1 − f (0, 0) = 1 es m´ınimo local y absoluto.

p 3

x2 + y 2

Observaci´ on: Si las desigualdades son v´alidas en todo U se tendr´a extremo absolutos. Punto Cr´ıtico: (Condici´ on necesaria) Definici´ on. Un punto del dominio de f es un punto cr´ıtico si todas las derivadas parciales de f son cero en el punto o f no es diferenciable en el punto. Caso particular: Si f : R2 → R, (x0 , y0 ) es punto cr´ıtico de f si y solo si :

i)

∂f (x0 , y0 ) = 0, ∂x 228

∂f (x0 , y0 ) =0 ∂y

o´ ii) ∄

Ejemplo :

∂f (x0 , y0 ) ∂f (x0 , y0 ) , y/o ∄ ∂x ∂y

Sea z = f (x, y) = x2 − y 2 =⇒

∂f ∂f = 2x = −2y ∂x ∂y

=⇒ (0, 0) punto cr´ıtico.

Punto silla: Definici´on : Sea punto (x0 , y0 ) ∈ Dom(f ) . Si cualquier Bδ (x0 , y0 ) contiene puntos (x, y) ∈ Bδ (x0 , y0 ) tales que f (x, y)− f (x0 , y0 ) > 0 y puntos (x, y) ∈ Bδ (x0 , y0 ) tales que f (x, y)− f (x0 , y0 ) < 0 se llama punto ensilladura. Nota. Un punto cr´ıtico en que f no es m´aximo ni m´ınimo se llama punto silla. Ejemplo: En la funci´on f (x, y) = x2 − y 2 , (0, 0) es punto ensilladura, pues ∆f = f (0, 0) − f (x, y) = −x2 + y 2 no se puede decir nada a´ un del 2 signo, pero para ∆f = f (0,0) − f (x, 0) = −x ≤ 0 y ∆f = f (0, 0) − f (0, y) = y 2 ≥ 0, permite concluir que (0, 0) es punto ensilladura.

Observaci´ on: Las definiciones anteriores son extensibles a funciones de m´as variables.

Teorema 3.3.8. Sea f una funci´ on continua de dos variables definida en una regi´on cerrada y acotada R del plano XY. Entonces: a.) Al menos hay un punto (x0 , y0 ) ∈ R en que f alcanza su valor m´ınimo. b.) Al menos hay un punto (x0 , y0 ) ∈ R en que f alcanza su valor m´ aximo. Observaci´ on: Este Teorema es extensible a m´as variables. Definici´ on: Sea f : U ⊆ Rn → R funci´on definida en un conjunto abierto U ∈ Rn y sea x ∈ U . Suponga que todas las derivadas parciales de segundo orden existen en x. A la matriz cuadrada de orden n. 229

A = (aij )i,j=1,...,n donde aij =

∂ 2f (x) ∂xi ∂xj

x = (x1 , x2 , ..., xn ) se llama matriz Hessiana (o simplemente Hessiano) de la funci´on f en x y se denota H(x). Caso particular: Para f (x, y) ∂2f ∂x2 ∂2f ∂x∂y

H(x, y) =

∂2f ∂y∂x ∂2f ∂y 2

!

Caso particular: Para f (x, y, z) 

 H(x, y, z) = 

∂2f ∂x2 ∂2f ∂x∂y ∂2f ∂x∂z

∂2f ∂y∂x ∂2f ∂y 2 ∂2f ∂y∂z

∂2f ∂z∂x ∂2f ∂z∂y ∂2f ∂z 2

  

Definici´ on. Sea la matriz 

  An =  

a11 a12 . . . a1n . a21 . . .. .. . . an1 ann

    

Consideremos las submatrices Ak de An (k = 1, 2, . . . , n) definidas de las siguientes maneras:     a11 a12 a13 a11 a12 A1 = (a11 ) , A2 = , A3 =  a21 a22 a23  , · · · , a21 a22 a31 a32 a33   a11 a12 . . . a1n .    a21 . .  An =  .  . ..  ..  an1 ann 230

Teorema 3.3.9. (Consideraciones suficientes para la existencia de extremos locales) Sea f : U ⊆ Rn → R una funci´ on definida en un conjunto abierto U de → − n R y x un punto cr´ıtico de f (es decir ∇f (x) = 0 ) y supongamos que las derivadas parciales de segundo orden son continuas en una vecindad abierta de x. Entonces: a.) Si todas las submatrices Ak de la matriz Hessiana H(x) definidas de la forma anterior tienen determinante positivo, f tiene un m´ınimo local en x. b.) Si todas las submatrices Ak de la matriz Hessiana H(x), definidas de la forma anterior, tienen determinantes de signo alternado comenzando por A1 < 0, f tiene un m´ aximo local en x.

Ejemplo 1: Sea f : R2 → R definida por f (x, y) = x2 + 3y 2 − 2x − 12y + 13. Determine los valores extremos de f . Soluci´ on Aplicando la condici´on necesaria de punto cr´ıtico, obtenemos ∇f (x, y) = (2x−2, 6y −12) = (0, 0) =⇒ (1, 2) es el u ´nico punto cr´ıtico. Calculemos la matriz Hessiana   2 0 H(x, y) = 0 6

Examinemos los determinantes de las submatrices A1 = (2) =⇒ det A1 = 2 > 0   2 0 =⇒ det A2 = 12 > 0 A2 = 0 6

∴ f tiene m´ınimo local en (1, 2).

Ejemplo 2: Sea f : R2 → R definida por f (x, y) = (x2 + 3y 2 ) e1−(x ¿Cu´ales son los puntos cr´ıticos de f ? Soluci´ on. De la condici´on de punto cr´ıtico tenemos: 2 2 2 2 ∂f = 2xe1−(x +y ) − 2x (x2 + 3y 2 ) e1−(x +y ) = 0 =⇒ ∂x 231

2 +y 2

).

∂f ∂x

= e1−(x

∂f ∂y

= 6ye1−(x

∂f ∂y

= e1−(x

2 +y 2

) (2x − 2x3 − 6xy 2 ) = 0

2 +y 2

2 +y 2

) − 2y (x2 + 3y 2 ) e1−(x2 +y2 ) = 0 =⇒

) (6y − 6y 3 − 2x2 y) = 0

De donde obtenemos, el sistema de ecuaciones   x (1 − x2 − 3y 2 ) = 0 x − x3 − 3xy 2 = 0 =⇒ =⇒ y (3 − 3y 2 − x2 ) = 0 3y − 3y 3 − x2 y = 0

Puntos Cr´ıticos: (0, 0), (0, 1), (0, −1), (1, 0), (1, 0), (−1, 0) Determine en qu´e puntos hay m´aximos, m´ınimos o puntos sillas.

y Ejemplo 3: Sea f : R4 − {0, 0, 0, 0} → R y f (x, y, z, u) = x + + x z u 1 + + y z u Determine los valores extremos de f. Soluci´on Determinemos los puntos cr´ıticos de la funci´on f ∂f ∂x ∂f ∂y ∂f ∂z ∂f ∂y

y =0 x2 1 z = − 2 =0 x y 1 u = − 2 =0 y z 1 1 = − 2 =0 z u = 1−

=⇒ (1, 1, 1, 1) Punto Cr´ıtico. La matriz Hessiana est´a definida como 

  H(x, y, z, u) =   

∂2f ∂x2 ∂2f ∂x∂y ∂2f ∂x∂z ∂2f ∂x∂u

232

∂2f ∂y∂x ∂2f ∂y 2 ∂2f ∂y∂z ∂2f ∂y∂u

∂2f ∂z∂x ∂2f ∂z∂y ∂2f ∂z 2 ∂2f ∂z∂u

∂2f ∂u∂x ∂2f ∂u∂y ∂2f ∂u∂z ∂2f ∂u2

     



 H(x, y, z, u) =  

2y x3 − x12

0 0

− x12 2z y3 − y12

0



0 − y12 2u z3 − z12

 0 0   − z12  2 u3

 2 −1 0 0  −1 2 −1 0   H(1, 1, 1, 1) =   0 −1 2 −1  0 0 −1 2 △1 = 2, △2 = 3, △3 = 2, △4 = 5

∴Hay un m´ınimo local en (1, 1, 1, 1). Teorema 3.3.10. (Condiciones suficientes para la existencia de extremos locales en dos variables.) Sea f : U ⊆ R2 → R una funci´on de dos variables que tiene derivadas parciales de segundo orden continuas en el conjunto abierto U. Sea fxx fxy (x, y) ∀ (x, y) ∈ U. Sea (x0 , y0 ) ∈ U tal det(H (x, y)) = fyx fyy que ∇f (x0 , y0 ) = 0. Entonces

i) Si det(H (x, y)) > 0 y fxx (x0 , y0 ) < 0, f tiene una m´ aximo local en (x0 , y0 ) .

ii) Si det(H (x, y)) > 0 y fxx (x0 , y0 ) > 0, f tiene un punto silla en (x0 , y0 ) . iii) Si det(H (x, y)) = 0 no podemos concluir nada.

3.3.7.

Extremos Restringidos

Pensemos, que se tiene una funci´on f (x, y) sujeta a cierta condici´on g(x, y) = 0. Se quiere maximizar o minimizar f (x, y) con la condici´on que (x, y) satisfaga la ecuaci´on g(x, y) = 0. En este caso podemos elaborar una respuesta aplicando la condici´on necesaria de valor extremo para f (x, y) ∂f ∂f dy df = + · =0 dx ∂x ∂y dx 233

dy Aplicando el teorema de la funci´on impl´ıcita podemos calcular a dx partir de la restricci´on: g(x, y(x)) = 0 =⇒

=⇒

∂g ∂g dy + · =0 ∂x ∂y dx

∂g dy = − ∂x ∂g dx ∂y

Sustituyendo en la primera ecuaci´on tenemos: df ∂f ∂f = + · dx ∂x ∂y =⇒



∂g ∂x ∂g ∂y

!

=0 ,

∂g 6= 0 ∂y

∂f ∂g ∂f ∂g + · =0 ∂x ∂y ∂y ∂x =⇒

∂(f, g) =0 ∂(x, y)

Lo que implica que los puntos cr´ıticos deben cumplir dos condiciones:

g(x, y) = 0. ∂(f, g) = 0 ∂(x, y) Adem´as, la decisi´on de valor extremo resulta del signo de

d2 f en cada dx2

punto cr´ıtico, ya que f (x, y) = f (x, y(x)) = f (x). Ejemplo 1. Obtenga las dimensiones de un rect´angulo de per´ımetro dado que determinan la mayor y la menor a´rea de ´este. Soluci´ on. Si x e y denotan las longitudes de los lados del rect´angulo, el problema consistir´a en buscar los extremos de la funci´on f (x, y) = xy, 234

x, y > 0

sujeta a la restricci´on g(x, y) = x+y − L2 = 0 , donde L es el per´ımetro dado.

Entonces aplicando las condiciones necesaria para obtener los puntos cr´ıticos tenemos: L g(x, y) = x + y − = 0.  2 ∂(f, g) y x = =⇒ y = x 1 1 ∂(x, y) Sustituyendo el resultado de la segunda ecuaci´on en la primera, se tiene: x=y=

L 4

Luego hemos encontrado que la funci´on f tiene un u ´nico punto cr´ıtico en   L L , . P = 4 4

Adem´as, determinemos la naturaleza del punto usando el criterio de la segunda derivada df dy =y+x dx dx L dy = −1 g(x, y) = x + y − = 0 =⇒ 2 dx df ∴ =y−x dx dy d2 f = − 1 = −2 < 0 2 dx dx

f (x, y (x)) = xy =⇒

2

Por tanto, en el punto ( L4 , L4 , L16 ) hay un m´aximo relativo sobre la curva de intersecci´on de las dos superficies z = xy y x + y =

L 2

La situaci´on se puede generalizar para funciones de m´as variables o m´as condiciones. 235

Por otra parte, para resolver este mismo tipo de problemas de m´aximo y m´ınimo podemos utilizar otro m´etodo que se fundamenta en el siguiente teorema. Teorema 3.3.11. (Multiplicadores de Lagrange) Sean f : U ⊆ Rn → R, g : U ⊆ Rn → R funciones de C 1 . Sean x0 ∈ U tal que g(x0 ) = 0 y S = {x ∈ U : g(x) = 0} supongamos adem´as que ∇g(x0 ) 6= 0. Si f |S (f restringida a S) tiene un m´ aximo o m´ınimo local en S, en x0 ,entonces existe λ ∈ R tal que: ∇f (x0 ) + λ∇g(x0 ) = 0 Nota: Lo anterior significa que x0 es un punto cr´ıtico de f |S .

M´etodo: Construir F (x, y, λ) = f + λg y determinar puntos cr´ıticos de F Ejemplo 1. Utilice el procedimiento que se origina a partir de este teorema para obtener el m´aximo local del ejemplo anterior. Soluci´ on. En este caso f (x, y) = xy,

x, y > 0

sujeta a la condici´on: g(x, y) = x + y −

L =0 2

. Sea F (x, y, λ) = xy + λ(x + y − L2 ), entonces Fx = y + λ = 0 Fy = x + λ = 0 L Fλ = x + y − = 0 2 de aqu´ı x = −λ, y = −λ y sustituyendo en la tercera ecuaci´on: −λ − λ −

L 2

= 0 =⇒ λ = − L4 .

El punto cr´ıtico es (x0 , y0 ) = (

L L , ). 4 4

2

El punto ( L4 , L4 , L16 ) es el m´aximo relativo sobre la curva de intersecci´on de las dos superficies z = xy y x + y = 236

L 2

x2 y 2 Ejemplo 2. De todos los rect´angulos inscritos en la elipse 2 + 2 = a b 1, a > 0 y b > 0 con lados paralelos al los ejes, determine el de mayor a´rea. Soluci´ on. En este caso f (x, y) = 4xy,

x, y > 0

x2 y 2 + 2 −1=0 a2 b   2 y2 x + 2 − 1 = 0 , entonces Sea F (x, y, λ) = 4xy + λ a2 b y g(x, y) =

x 2a2 y = 0 =⇒ λ = − a2 x y 2b2 x = 4x + 2λ 2 = 0 =⇒ λ = − b y 2 2 y x = 2 ∴ 2 a b

Fx = 4y + 2λ Fy

Reemplazando esta u ´ltima expresi´on en la ecuaci´on x2 y 2 Fλ = 2 + 2 − 1 = 0 =⇒ a b 

 a b obtenemos a un u ´nico punto cr´ıtico: √ , √ 2 2 Adem´as, determinemos la naturaleza del punto usando el criterio de la segunda derivada f (x, y (x))

= =⇒

g(x, y)

= =⇒

dy df = 4y + 4x dx dx dy d2 f d2 y = 4 + 4x 2 dx2 dx dx 2 2 x y dy b2 x + − 1 = 0 =⇒ = − a2 b2 dx a2 y   d2 y b2 1 b2 x 2 = − + dx2 a2 y a2 y 3

4xy =⇒

237

duce



 a b Evaluando segunda derivada en el punto P √ , √ , pro2 2

  d2 f b2 x b b4 x 3 d2 f =⇒ = −8 = −12 < 0 − 4 2 2 4 3 2 dx ay a y dx a   a b √ ,√ Por tanto, el punto hay m´aximo relativo cuyo valor 2 2   a b = 2ab es f √ , √ 2 2 Ejemplo 3: Obtenga los extremos posibles de f (x, y) = x2 + 24xy + 8y 2 con la restricci´on x2 + y 2 − 25 = 0.

Soluci´ on:

Sea F (x, y, λ) = x2 + 24xy + 8y 2 + λ (x2 + y 2 − 25) . Fx = 2x + 24y + 2λx = 0 =⇒ (1 + λ)x + 12y = 0 (1) Fy = 24x + 16y + 2λy = 0 =⇒ 12x + (8 + λ)y = 0 (2) Fλ = x2 + y 2 − 25 = 0 (3) Multiplicando la ecuaci´on (1) por 12 y (3) por(1 + λ) produce.

12(1 + λ)x + 144y = 0 12(1 + λ)x + (8 + λ)(1 + λ)y = 0

Restando ambas ecuaciones =⇒ λ2 + 9λ − 136 = 0 =⇒ λ1 = 8; λ2 = −17  18x + 24y = 0 =⇒ 3x + 4y = 0 =⇒ y = − 34 x Si λ = 8 =⇒ 24x + 32y = 0 2 x2 + 34 x = 25 =⇒ x = ±4 Lo que da los puntos cr´ıticos (±4, ∓3) 238

Si λ = −17 en forma similar se obtiene que (±3, ±4) son puntos cr´ıticos. Localizaci´ on del m´ aximo y el m´ınimo absoluto de f sobre un dominio compacto. Para determinar m´aximos y m´ınimos absolutos e una regi´on cerrada y acotada encerrada por una curva suave, se debe: •

Determinar todos los puntos cr´ıticos en el interior de la regi´on.



Evaluar f en los puntos, cr´ıticos.



Usar Lagrange para determinar puntos cr´ıticos en la frontera.

• Comparar todos estos valores y seleccionar el m´aximo y el m´ınimo.

Criterio de la Segunda Derivada para Extremos Restringidos. Sea f (x, y) funci´on a maximizar o minimizar y g(x, y) = 0 la condici´on: S = {(x, y) ∈ R2 | g(x, y) = 0} .Sea F (x, y, λ) = f (x, y) + λg(x, y) Sea (x0 , y0 , λ) punto cr´ıtico de F

∂g +λ ∂x |(x0 ,y0 ) = 0 ∂g +λ ∂y |(x0 ,y0 ) = 0 g (x0 , y0 ) = 0

∂f | ∂x (x0 ,y0 ) ∂f | ∂y (x0 ,y0 )

Se desea utilizar el criterio de la segunda derivada para identificar extremos locales de la funci´on f (x, y) bajo la condici´on g (x, y) = 0 lo que conduce encontrar los m´aximos y m´ınimos de funciones de una variable f (x, y(x)) . De la condici´on, calculemos y = y(x) y reemplacemos en f |S . Aqu´ı podemos considerar f como funci´on de una variable, es decir f (x, y) = f (x, y(x)) en S. Nos proponemos calcular

∂2f ∂x2

∂f ∂f dy df = + · dx ∂x ∂y dx 239

 2 ∂ 2f ∂ 2 f dy ∂ 2 f dy ∂ 2 f dy ∂f d2 y d2 f = + · · + · + · + dx2 ∂x2 ∂y∂x dx ∂x∂y dx ∂y 2 dx ∂y dx2  2 ∂ 2f ∂ 2 f dy ∂ 2 f dy ∂f d2 y = + 2 · · + · + ∂x2 ∂y∂x dx ∂y 2 dx ∂y dx2 g(x, y(x)) = 0 =⇒

=⇒

∂g ∂g dy + · =0 ∂x ∂y dx

∂g dy = − ∂x ∂g dx ∂y

Derivando nuevamente g respecto de x

∂ 2g ∂ 2 g dy ∂ 2 g dy ∂ 2 g + · + · + · ∂x2 ∂y∂x dx ∂x∂y dx ∂y 2

Sustituyendo

2



dy dx

2

+

∂g d2 y · =0 ∂y dx2

dy se tiene: dx

2

∂ g ∂ g +2 · 2 ∂x ∂y∂x



∂g ∂x ∂g ∂y

!

2

∂ g + 2· ∂y

∂ 2 g ∂g ∂ 2 g ∂g ∂ 2 g · · − 2 · + ∂x2 ∂y ∂y∂x ∂x ∂y 2





∂g ∂x ∂g ∂y

∂g ∂x

!2

2

∂g d2 y =0 /· · + ∂y dx2

1

·  2 + ∂g ∂y



∂g ∂y

2

·



∂g ∂y



d2 y =0 dx2

" #  2 d2 y ∂ 2 g ∂g 1 ∂ 2 g ∂g ∂ 2 g ∂g 1 =⇒ 2 =  2 − 2 · +2 · − · · ∂g dx ∂x ∂y ∂y∂x ∂x ∂y 2 ∂x ∂g ∂y ∂y

Reemplazando en

d2 f dx2

240

! !2 ∂g ∂g 2 ∂ 2f ∂ 2f d2 y ∂ f = −2 · ∂x + 2 · ∂x ∂g ∂g dx2 ∂x2 ∂y∂x ∂y ∂y ∂y " #  2 1 ∂ 2 g ∂g ∂ 2 g ∂ 2 g ∂g ∂g 1 ∂f · ·  2 − 2 · +2 · − · ∂g + ∂y ∂x ∂y ∂y∂x ∂x ∂y 2 ∂x ∂g ∂y ∂y

y0 = y(x0 ),

∂f ∂g ∂f ∂g |x0 = −λ | x0 ; |x0 = −λ |x ∂x ∂x ∂y ∂y 0

Luego

(x0 , y0 ) ,

∂f | ∂x x0 ∂g | ∂x x0

= −λ,

∂f ∂y ∂g ∂y

| x0

| x0

= −λ

"  2  2 ∂ 2f d2 f 1 ∂g ∂g ∂ 2 f ∂g ∂g ∂ 2 f =  2 · · · + 2 · −2 2 2 dx ∂x ∂y ∂y∂x ∂x ∂y ∂y ∂x ∂g ∂y

 2  2 # ∂ 2g ∂ 2g ∂g ∂g ∂ 2 g ∂g ∂g +λ 2 · · · +λ 2 · − 2λ ∂x ∂y ∂y∂x ∂x ∂y ∂y ∂x "      2 ∂ 2f 1 ∂ 2g ∂g ∂ 2g ∂ 2f =  2 +λ 2 · +λ · −2 ∂x2 ∂x ∂y ∂y∂x ∂y∂x ∂g ∂y

∂g ∂g · · + ∂x ∂y



∂ 2g ∂ 2f + λ ∂y 2 ∂y 2

  2 # ∂g · ∂x

Podemos escribir entonces: "  2  2 # 1 ∂g ∂ 2 F ∂g ∂g ∂ 2 F ∂g ∂ 2F d2 f =  2 · · · · + −2 2 2 2 dx ∂x ∂y ∂y∂x ∂x ∂y ∂y ∂x ∂g ∂y

En el par´entesis, cuadrado se tiene 241

∂g ∂g ∂g ∂g ∂x ∂g ∂y ∂y ∗ = ∂2F ∂2F + ∂2F ∂x ∂y∂x ∂y2 ∂y ∂y∂x ∂g ∂g 0 0 ∂x ∂y ∂g ∂g ∂ 2 F ∂ 2 F = − ∂x ∂x2 ∂y∂x = − ∂x ∂g ∂g ∂ 2 F ∂ 2 F ∂y ∂y ∂y∂x ∂y2

De lo anterior se plantea la siguiente definici´on

∂g ∂x ∂2F ∂x2



∂g ∂x ∂2F ∂x2 ∂2F ∂y∂x

∂g ∂y ∂2F ∂y∂x ∂2F ∂y 2



Si f y g son funciones como las definidas antes y F = f +λg. Llamamos hessiano limitado de la funci´on F a ∂g ∂g 0 ∂x ∂y ∂g ∂ 2 F ∂ 2 F HF = ∂x en (x0 , y0 ) 2 ∂g ∂∂x2 F ∂y∂x 2F ∂ ∂y ∂y∂x ∂y2 De tal modo que

HF (x0 , y0 ) d2 f = − 2 2 dx ∂g (x , y ) 0 0 ∂y Teorema en extremos restringidos Consideremos nuevamente d2 f HF (x0 , y0 ) = − 2 2 dx ∂g (x , y ) 0 0 ∂y 2

De esta expresi´on, dado que el signo de la segunda derivada ddxf2 depende  2 ∂g s´olo del determinante Hessiano limitado, ya que ∂y (x0 , y0 ) > 0, inferimos el siguiente criterio de la segunda derivada para extremos restringidos. a) Si HF (x0 , y0 ) > 0, entonces local condicionado en (x0 , y0 ).

d2 f dx2

242

< 0 y la funci´on tiene un m´aximo

b) Si HF (x0 , y0 ) < 0, entonces local condicionado en (x0 , y0 ) .

d2 f dx2

> 0 y la funci´on tiene un m´ınimo

c) Si HF (x0 , y0 ) = 0 no hay informaci´on del punto(x0 , y0 ).

Ejemplo: Hallar los extremos de f (x, y) = (x − y)2 sujeta a la restricci´on x2 + y 2 − 1 = 0

Soluci´ on:

Sea F (x, y, λ) = (x − y)2 + λ (x2 + y 2 − 1)  Fx = 2 (x − y) + 2λx = 0 =⇒ 2λ (x + y) = 0 Fy = −2 (x − y) + 2λy = 0 F λ = x2 + y 2 = 1

a) Si escogemos λ = 0 , y (x + y) 6= 0 obtenemos los puntos cr´ıticos de f libre de restricci´on. b) Si λ 6= 0 =⇒ y = −x. Reemplazando en la tercera ecuaci´on , se obtienen dos puntos cr´ıticos de la funci´on f sujetos a la restricci´on.     1 1 √1 1 √ √ √ P1 2 , − 2 y P 2 − 2 , 2

Determinemos el Hessiano l´ımitado para decidir la naturaleza de estos puntos cr´ıticos 0 2x 2y −2 HF (x, y) = 2x 2 + 2λ 2y −2 2 + 2λ   = −8 x2 + 2xy + y 2 − 8λ x2 + y 2 Sustituyendo y = −x en la primera ecuaci´on se tiene λ = −2 6= 0,entonces el Hessiano se reduce a   HF (x, y) = −8 x2 + 2xy + y 2 + 16 x2 + y 2  = 8 x2 − 2xy + y 2

Evaluando en el primer punto cr´ıticos produce 243

HF



1 1 √ , −√ 2 2



= 16 > 0

   d2 f  1 1 1 1 √ √ √ √ entonces , − 2 < 0, luego en el punto , − 2 la funci´on 2 2 dx2 f tiene un m´aximo local condicionado. Por simetr´ıa, el segundo punto tambi´en corresponde a un m´aximo local condicionado de f. El criterio de la segunda derivada para extremos restringidos se puede generalizar para funciones de m´as de dos variables. En el caso de una funci´on de tres variables x, y, z y sujeta a la sola restricci´on g(x, y, z) = 0, formamos el Hessiano correspondiente a F (x, y, z, λ) = f (x, y, z) + λg (x, y, z) de la siguiente forma 0 ∂g HF = ∂x ∂g ∂y ∂g

∂g ∂x ∂2F ∂x2 ∂2F ∂y∂x ∂2F ∂x∂z

∂z

y sea

0 ∂g A3 = ∂x ∂g ∂y

∂g ∂x ∂2F ∂x2 ∂2F ∂y∂x

∂g ∂y ∂2F ∂y∂x ∂2F ∂y 2 ∂2F ∂y∂z

∂g ∂y ∂2F ∂y∂x ∂2F ∂y 2

∂g ∂z ∂2F ∂x∂z ∂2F ∂y∂z ∂2F ∂z 2





El criterio de la segunda derivada en este caso expresa: Sea (x0 , y0 , z0 ) punto cr´ıtico. a) Si HF < 0 y A3 > 0,entonces la funci´on f tendr´a un m´aximo condicionado en (x0 , y0 , z0 ). b) Si HF < 0 y A3 < 0,entonces la funci´on f tendr´a un m´ınimo condicionado en (x0 , y0 , z0 ). c) Si HF > 0,entonces la funci´on f no tiene extremos condicionado en (x0 , y0 , z0 ). d.) Si HF = 0, no hay informaci´on acerca del punto (x0 , y0, z0 ). 244

Ejemplo Hallar los extremos de la funci´on f (x, y, z) = x2 + y 2 + z 2 sujeta a la restrici´on g (x, y, z) = z 2 + 2x − y 2 − 1 = 0. Soluci´ on:

Sea la funci´on F = x2 + y 2 + z 2 + λ (z 2 + 2x − y 2 − 1) Fx Fy Fz Fλ

= = = =

2x + 2λ = 0 =⇒ 2 (x + λ) = 0 =⇒ x = −λ 2y − 2λy = 0 =⇒ 2y(1 − λ) = 0 =⇒ y = 0 2z + 2λz = 0 =⇒ 2z(1 + λ) = 0 =⇒ z = 0 z 2 + 2x − y 2 − 1 = 0

Reemplazando en Fλ : 02 +2(−λ)−02 −1 = 0 =⇒ λ = − 21 , y obtenemos el punto cr´ıtico 21 , 0, 0 HF

0 2 −2y 2z 2 2 0 0 = −2y 0 2 − 2λ 0 2z 0 0 2 + 2λ  = −4 4 − 4x2 − 2y · 2 · 2y (2 − 2λ) + 2z · 2 (2 − 2λ) (−2z)   1 1 , 0, 0, − = −16 < 0 HF 2 2 A3 = 8y 2 − 4 (2 − 2λ) = 0 − 12 < 0

∴Hay un m´ınimo condicionado de f en el punto

1 , 0, 0 2



.

Obs´ervese que las otras opciones no producen puntos cr´ıticos en IR3

3.4.

Problemas Resueltos

3.4.1.

Continuidad y diferenciabilidad

Problema Dada la funci´on f : R2 → R definida como ( xy arctg 2 , si (x, y) 6= (0, 0) x + y2 f (x, y) = . 0 , si (x, y) = (0, 0) 245

a) Verificar si f es continua en IR2 b) Calcular si existen las derivadas parciales

∂f ∂f , en IR2 ∂x ∂y

Soluci´on. a) Tenemos que f (x, y) es continua ∀ (x, y) 6= (0, 0) puesto que es xy composici´on de dos funciones continuas, como son arctg y 2 . x + y2 Para estudiar la continuidad en el punto (0, 0) tenemos que calcular l´ım

(x,y)→(0,0)

entonces

f (x, y) lo que haremos a trav´es de la trayectoria y = mx, l´ım f (x, mx) = l´ımarctg x→0

(x,y)→(0,0)

m mx2 = l´ımarctg 2 2 x→0 x +y 2 + m2

que depende de la pendiente m, por lo que este l´ımite no existe. Por lo tanto f no es continua en el punto(0, 0) b) Para (x, y) 6= (0, 0) la funci´on admite derivadas parciales, que son: ∂f y 2 − x2 , (x, y) = y 2 ∂x (x + y 2 )2 + x2 y 2 x2 − y 2 ∂f (x, y) = x 2 ∂y (x + y 2 )2 + x2 y 2

Para (x, y) = (0, 0) ,se tiene f (h, 0) − f (0, 0) h arctg0 − 0 = l´ım = l´ım0 = 0 h→0 h→0 h

∂f (0, 0) = ∂x

l´ım

h→0

Analogamente, derivando con respecto a y f (0, h) − f (0, 0) h→0 h arctg0 − 0 = l´ım = l´ım0 = 0 h→0 h→0 h

∂f (0, 0) = ∂y

l´ım

Por lo tanto, existen las derivadas parciales en (x, y) = (0, 0) . 246

Problema Dada la funci´on f : IR2 → IR definida como  2  x seny 2 , si (x, y) 6= (0, 0) f (x, y) = , probar que es x2 + y 2  0 , si (x, y) = (0, 0)

diferenciable en el punto P0 = (0, 0) .¿Es continua la funci´on en ese punto? Soluci´ on. Tenemos que utilizar la definici´on y ver si el siguiente l´ımite es cero: L=

l´ım

(h,k)→(0,0)

|∆f − df | √ h2 + k 2

h2 senk 2 ,y h2 + k 2

donde ∆f = f (h, k) − f (0, 0) = ∂f ∂f (0, 0) h + (0, 0) k ∂x ∂y En este caso

df =

f (h, 0) − f (0, 0) h→0 h 2 h ·0 −0 2 =0 = l´ım h h→0 h

∂f (0, 0) = ∂x

l´ım

Asimismo f (0, k) − f (0, 0) h→0 k 0 · senk 2 −0 2 k = l´ım =0 h→0 k

∂f (0, 0) = ∂y

Luego, df = 0, entonces L =

l´ım

h2 senk 2 √ (h,k)→(0,,0) (h2 + k 2 ) h2 + k 2 l´ım

h2 senk 2

h2 k 2

≤ (h2 + k 2 )3/2 (h2 + k 2 )3/2 (h2 + k 2 )(h2 + k 2 ) p 2 ≤ = (h + k 2 ) < ε 3/2 2 2 (h + k )

g (h, k) =

247

Si δ = ε . As´ı L = 0 y f es diferenciable en P0 = (0, 0) . De lo anterior se deduce que f es es continua en (0, 0) ya que es diferenciable en dicho punto.

3.4.2.

Regla de la cadena

Problema 1 Sea la ecuaci´on zxx + 2zxy + zy = 0 , donde

x(u, v) =

u−v u2 − v 2 u+v , y(u, v) = , z(u, v) = − w (u, v) 2 2 4

Muestre que al cambiar las variables independientes (x, y) por (u, v) y la funci´on z por w la ecuaci´on se reduce a 2 − 4wuu = 0.

Soluci´ on.

En primer lugar, calculamos la aplicaci´on inversa u(x, y) = x + y, v (x, y) = x − y. Derivando parcialmente estas u ´ltimas expresiones se tiene: ux = 1, uy = 1; vx = 1, vy = −1

Usando estos resultados y la regla de la cadena, obtenemos z x = z u ux + z v vx = z u + z v z y = z u uy + z v vy = z u − z v

Reiterando la derivaci´on parcial y usando la regla de la cadena por segunda vez zxx = (zx )u ux + (zx )v vx = zuu + zvu + zuv + zvv

zxy = (zx )u uy + (zx )v vy = zuu + zvu − (zuv + zvv )

zyy = (zx )u uy + (zv )v vy = zuu − zvu − (zuv − zvv )

Suponiendo que z es una funci´on continua con primeras derivadas parciales 248

continuas, entonces zxx + 2zxy + zy = 4zuu = 0 1 2u − wu =⇒ zuu = − wuu 4 2 Por tanto, se tiene 1 − wuu = 0 2 Finalmente, zu =

Problema 2 Una funci´on z = z (x, y) se dice que es arm´onica si tiene derivadas parciales de segundo orden continuas y adem´as zxx + zyy = 0. x y Sean u = 2 , v= 2 . Pruebe que: 2 x +y x + y2 i) u y v son arm´onicas ii) (ux )2 = (vy )2 iii) (uy )2 = (vx )2 iv) ux vx = −uy vy b)Si f (x, y) es unafunci´on arm´onica, entonces la funci´on w (x, y) = y x , 2 f 2 2 x + y x + y2 es tambi´en arm´onica Soluci´ on. a) i) u = v=

x y 2 − x2 2xy =⇒ u = , uy = − 2 x 2 2 2 2 2 x +y (x + y ) (x + y 2 )2

y 2xy x2 − y 2 =⇒ v = − , v = x y x2 + y 2 (x2 + y 2 )2 (x2 + y 2 )2

Derivando parcialmente por segunda vez se tiene −2x(x2 + y 2 )2 − (y 2 − x2 )2(x2 + y 2 )2x (x2 + y 2 )4 2x3 − 6xy 2 = (x2 + y 2 )3

uxx =

249

−2x(x2 + y 2 )2 − (2xy)2(x2 + y 2 )2y (x2 + y 2 )4 −2x3 + 6xy 2 = (x2 + y 2 )3

uyy =

Lo anterior implica que uxx + uyy = 0 Analogamente para , vxx + vyy = 0 Adem´as



2 x2 − y 2 ii) (ux ) = = = (vy )2 (x2 + y 2 )2  2  2 2xy 2xy 2 iii)(uy ) = − 2 = = (vx )2 2 2 2 2 2 (x + y ) (x + y )  2     2 y −x 2xy 2xy(y 2 − x2 ) iv) ux vx = − 2 =− = −uy vy (x2 + y 2 )2 (x + y 2 )2 (x2 + y 2 )2 2

y 2 − x2 (x2 + y 2 )2

2



b) Aplicando derivaci´on compuesta tenemos:

∂f ∂f ux + vx , ∂u ∂v ∂f ∂f uy + vy = ∂u ∂v

wx = wy

Derivando parcialmente por segunda vez, queda wxx

wyy



 ∂f ∂ 2f ∂ 2f = ux + v x ux + uxx + 2 ∂u ∂v∂u ∂u  2  ∂ f ∂f ∂ 2f + ux + 2 v x v x + vxx ∂u∂v ∂v ∂v   2 ∂f ∂ 2f ∂ f uy + v y uy + uyy + = 2 ∂u ∂v∂u ∂u  2  ∂ f ∂f ∂ 2f + uy + 2 v y v y + vyy ∂u∂v ∂v ∂v

Finalmente 250

∂ 2f ∂ 2f ∂f 2 (u ) + vx ux + uxx + x 2 ∂u ∂v∂u ∂u ∂ 2f ∂f ∂ 2f ux vx + 2 (vx )2 + vxx + ∂u∂v ∂v ∂v ∂ 2f ∂f ∂ 2f 2 (u ) + uy vy + uyy + = y 2 ∂u ∂v∂u ∂u ∂ 2f ∂f ∂ 2f + uy vy + 2 (vy )2 + vyy ∂u∂v ∂v ∂v

wxx =

wyy

Sumando t´erminos y utilizando las igualdades establecidas en a) se tiene wxx + wyy

3.4.3.

 ∂f ∂ 2f ∂ 2f + 2 (ux )2 + (uxx + uyy )+ = 2 ∂u ∂v ∂u   2 ∂ 2f ∂ f ∂f + 2 (vx )2 + (vxx + vyy ) = 0 + 2 ∂u ∂v ∂v 

Derivaci´ on Impl´ıcita

Problema 1 a) Sea f : R → R2 una funci´on tal que grad f (1, 1) = (2, 4) y g : R3 → R2 una funci´on tal que sus funciones coordenadas gi : R3 → R, i = 1, 2 tienen los siguientes gradientes grad g1 (1, 1, 1) = (2, 3, 1), ∂(f ◦ g) (1, 1, 1). grad g2 (1, 1, 1) = (−5, 4, 2). Si g(1, 1, 1) = (1, 1). Obtener ∂x b) Utilizando el teorema de la funci´on impl´ıcita determine si es posible 3 escribir y en t´erminos de x para la funci´on F (x, y) = x4 − exy −1 = 0 en una vecindad del punto (1, 1), y adem´as encuentre su derivada. Soluci´ on Sean las coordenadas cartesianas designadas en R3 por (x, y, z) y en R2 por (u, v), y tomando u = u(x, y, z), v = v(x, y, z) tenemos que: ∂f ◦ g ∂f ∂u ∂f ∂v (1, 1, 1) = (g(1, 1, 1)) · (1, 1, 1) + (g(1, 1, 1)) · (1, 1, 1) ∂x ∂u ∂x ∂v ∂x ∂u ∂f ∂v ∂f (1, 1) · (1, 1, 1) + (1, 1) · (1, 1, 1) = ∂u ∂x ∂v ∂x 251

Notemos que 

 ∂f ∂f grad f (1, 1) = (1, 1), (1, 1) = (2, 4), ∂u ∂v     ∂u ∂u ∂u ∂g1 ∂g1 ∂g1 grad g1 = = = (2, 3, 1), , , , , ∂x ∂y ∂z ∂x ∂y ∂z     ∂v ∂v ∂v ∂g2 ∂g2 ∂21 = = (−5, 4, 2), , , , , grad g2 = ∂x ∂y ∂z ∂x ∂y ∂z As´ı

∂f ◦ g (1, 1, 1) = 2 · 2 + 4 · −5 = −16. ∂x

Primeramente note que (1, 1) ∈ F −1 (0, 0) y adem´as ∂F ∂F 3 3 = 4x3 − y 3 exy −1 , = −3xy 2 exy −1 ∂x ∂y las cuales son continuas en R2 en particular para alguna vecindad V ((1, 1), δ) de (1, 1) donde ∂F (1, 1) = −3 6= 0 por lo tanto podemos ∂y ocupar el teorema de la funci´on impl´ıcita, y definir f : V ((1, 1), δ) → R con y = f (x) y 1 = f (1) cuya derivada es 3

4x3 − y 3 exy −1 y = . 3xy 2 exy3 −1 ′

Problema 2 a) Determine las derivadas parciales ∂u/∂x, ∂u/∂y, ∂v/∂x, ∂v/∂y, donde u, v son funciones definidas implicitamente por el sistema F (x, y, u, v) = xeu+v + uv − 1 = 0, G(x, y, u, v) = yeu−v − 2uv − 1 = 0. alrededor del punto p = (1, 1, 0, 0). ∂ 2z b) Sea la funci´on z = f (u + v , u/v) obtener . ∂u2 2

2

252

Soluci´ on. a) Verificando las hip´otesis del Teorema de la funci´on impl´ıcita podemos concluir que: ∂u (yeu−v + 2u)eu+v = − u+v u−v ∂x 2x ye + yeu−v u − 2vxeu+v + 2xeu+v u + vyeu−v eu+v (yeu−v − 2v) ∂v = − u+v u−v ∂x 2x ye + yeu−v u − 2vxeu+v + 2xeu+v u + vyeu−v ∂u e( u − v)(xeu+v + u) = − u+v u−v ∂y 2x ye + yeu−v u − 2vxeu+v + 2xeu+v u + vyeu−v ∂v (xeu+v + v)eu−v = u+v u−v ∂y 2x ye + yeu−v u − 2vxeu+v + 2xeu+v u + vyeu−v

b) Definiendo x = x(u, v) = u2 + v 2 , y = y(u, v) = u/v tenemos que z = f (x, y), entonces ∂z ∂x ∂z ∂y ∂z ∂z 1 ∂z = + = 2u + , ∂u ∂x ∂u ∂y ∂u ∂x ∂y v luego

    2 ∂ 2z 1 ∂ 2 z ∂y ∂z ∂ 2 z ∂y ∂ 2 z ∂x ∂ z ∂x + = 2 + 2u + + ∂u2 ∂x ∂x2 ∂u ∂y∂x ∂u v ∂y 2 ∂u ∂x∂y ∂u     2 1 ∂ 2z ∂ 2z 1 ∂ 2z 1 ∂ x ∂z + 2u + 2u + + 2u = 2 ∂x ∂x2 ∂y∂x v v ∂y 2 ∂x∂y v

Utilizando finalmente el teorema de Schwarz tenemos que 2 ∂ 2z ∂z 1 ∂ 2 z 4u ∂ 2 z 2∂ z = 2 + + + 4u . ∂u2 ∂x ∂x2 v ∂y 2 v ∂x∂y

Problema 3 a) Si u = f (x, y, z) define una funci´on diferenciable, y z se define implicitamente como una funci´on de x e y por la ecuaci´on g(x, y, z) = 0 con los atributos pedido en el teorema de la funci´on impl´ıcita. Pruebe que u tiene primeras derivadas parciales de x e y dadas por: ∂(f,g) ∂(f,g) ∂u ∂u ∂(x,z) ∂(y,z) = ∂y ; = ∂y ∂x ∂y ∂z ∂z 253

b) Si u = x2 y + z 2 , y z = g(x, y) se define impl´ıcitamente por la ecuaci´on x2 y − 3z + 8yz 3 = 0 Calcule: ∂u ∂u (1, 0, 0) y (1, 0, 0) ∂x ∂y Soluci´ on. a) Utilizando la regla de la cadena tenemos ∂f ∂f ∂z ∂u = + ∂x ∂x ∂z ∂x por otra parte si g(x, y, z) = 0 define impl´ıcitamente a z = z(x, y) entonces ∂g ∂z = − ∂x ∂g ∂x ∂z reemplazando en la ecuaci´on anterior ∂g ∂f ∂f ∂u ∂x = + (− ∂g ) ∂x ∂x ∂z ∂z

=

∂f ∂g ∂x ∂z



∂f ∂g ∂z ∂x

∂g ∂z

=

∂(f,g) ∂(x,z) ∂g ∂z

Similarmente ∂f ∂f ∂z ∂u = + ∂y ∂y ∂z ∂y

∂g

y

∂z ∂y = − ∂g ∂y ∂z

De lo anterior se deduce ∂g

∂u ∂f ∂f ∂y = + (− ∂g ) ∂y ∂y ∂z ∂z =

∂f ∂g ∂y ∂z

− ∂g ∂z

254

∂f ∂g ∂z ∂y

=

∂(f,g) ∂(y,z) ∂g ∂z

b) En este caso u = f (x, y, z) = x2 y + z 2 y z = z(x, y) se define impl´ıcitamente por g(x, y, z) = x2 y − 3z + 8yz 3 = 0 ,luego tenemos ∂g ∂g ∂g = 2xy , = x2 + 8z 3 , = −3 + 24yz 2 ∂x ∂y ∂z derivadas que son todas continuas por lo que se afirma que g es de C1 Adem´as g(1, 0, 0) = 0 y ∂g (1, 0, 0) = −3 6= 0 ∂z Entonces por el teorema de la funci´on impl´ıcita se tiene que existe V = Vδ (1, 0) y una vecindad (−a, a) de z = 0 y una funci´on z(x, y) de C1 sobre V tal que z(1, 0) = 0 y z(1, 0)ǫ(−a, a) Calculemos el Jacobiano

2xy ∂(f, g) 2z = 2 2xy −3 + 24yz ∂(x, z) = 2xy(−3 + 24yz 2 ) − 2xy2z = 2xy(−3 + 24yz 2 − 2z)

Por otra parte

∂g = −3 + 24yz 2 ∂z

Entonces ∂u 2xy(−3 + 24yz 2 − 2z) ⇒ = ∂x −3 + 24yz 2 0 ∂u (1, 0, 0) = =0 ∂x −3 Tambi´en: x2 ∂(f, g) 2z = 2 3 2 x + 8z −3 + 24yz ∂(y, z) = −3x2 + 24x2 yz 2 − 2x2 z − 16z 3 = x2 (24yz 2 − 2z − 3) − 16z 3 255

Finalmente x2 (24yz 2 − 2z − 3) − 16z 3 ∂u = ∂y −3 + 24yz 2 ∂u −3 ⇒ (1, 0, 0) = =1 ∂y −3 ⇒

3.4.4.

Plano Tangente a una Superficie

Problema 1 Probar que los planos tangentes a la superficie S: xyz = a, a > 0 constante, en cualquier punto de S forma con los planos coordenados un tetraedro de volumen constante. Soluci´ on. Sea la superficie S descrita por la funci´on impl´ıcita F (x, y, z) = xyz − a = 0. los vectores normales a la superficie S satisfacen ∇F (x, y, z) = (yz, xz, xy) → − → − Si P 0 = (x0 , y0 , z0 ) ∈ S,entonces N (P0 ) = ∇F (x0 , y0 , z0 ) = (y0 z0 , x0 z0 , x0 y0 ) → − La ecuaci´on del plano tangente al punto P 0 ∈ S, esta definida por → − → − − (→ r − P )· N (P ) = 0,luego y z (x − x )+x z (y − y )+x y (z − z ) = 0

0

0 0

0

0 0

0

0 0

0.

Entonces, las trazas de este plano con los ejes coordenados son 3x0 y0 z0 = 3x0 i) Si x = α, y = 0, z = 0 =⇒ α = y0 z 0 3x0 y0 z0 ii) Si x = 0, y = β, z = 0 =⇒ β = = 3y0 x0 z 0 3x0 y0 z0 = 3z0 iii) Si x = 0, y = 0, z = γ =⇒ α = x0 y0 El volumen del tetraedro es: αβγ (3x0 ) (3y0 ) (3z0 ) 9 V = = = a, constante. 6 6 2

256

0

Problema 2 a) Probar que S1 dada por F (x, y, z) = 0 y S2 dada por G(x, y, z) = 0 son ortogonales en sus puntos de intersecci´on s´ı y solo si Fx Gx + Fy Gy + Fz Gz = 0. b) Probar que las superficies S1 : x2 + y 2 + z 2 − 2x + 4y − 8z = 0 y S2 : 3x2 − y 2 + 2z 2 − 6x − 4y − 16z + 31 = 0 son ortogonales. Soluci´ on.

→ − a) La normal a S1 dada por F (x, y, z) = 0 es N 1 = ∇F (x, y, z) = (Fx , Fy , Fz ). Del mismo modo, la normal a S2 dada por G(x, y, z) = 0 es → − N 2 = ∇G(x, y, z) = (Gx , Gy , Gz ) Por definici´on, ambas superficies son ortogonales si ∇F · ∇G = 0 ⇐⇒ (Fx , Fy , Fz ) · (Gx , Gy , Gz ) = 0 =⇒ F x Gx + F y Gy + F z Gz = 0 b) Sea S1 : x2 + y 2 + z 2 − 2x + 4y − 8z = 0 =⇒ (x − 1)2 + (y + 2)2 + (z − 4)2 − 21 = 0 y S2 : 3x2 − y 2 + 2z 2 − 6x − 4y − 16z + 31 = 0 =⇒ 3(x − 1)2 − (y + 2)2 + 2(z − 4)2 = 0 Entonces S1 tiene normal → − N 1 = ∇F (x, y, z) = (2x − 2, 2y + 4, 2z − 8) Asimismo , S2 tiene normal → − N 2 = ∇G(x, y, z) = (6x − 6, −2y − 4, 4z − 16) =⇒ ∇F · ∇G = (2x − 2, 2y + 4, 2z − 8) · (6x − 6, −2y − 4, 4z − 16)

= (2x − 2) (6x − 6) + (2y + 4) (−2y − 4) + (2z − 8) (4z − 16)

= 4 [3(x − 1)2 − (y + 2)2 + 2(z − 4)2 ] = 4 · 0 = 0

Por lo tanto, la superficie S1 es ortogonal a S2 . 257

3.4.5.

Derivadas Direccionales

Problema 1   2xy 2 , si (x, y) 6= (0, 0) Sea f (x, y) : . Determine, si existe, la x2 + y 4  0 , si (x, y) = (0, 0)

derivada direccional de f en P0 = (0, 0). Soluci´ on.

Sea eb = (e1 , e2 ) un versor en una direcci´on cualquiera de IR2 tal que

f (λe1 , λe2 ) − f (0, 0) f ((0, 0) + λ (e1 , e2 )) − f (0, 0) = l´ım λ→0 λ→0 λ λ

Dbe f (0, 0) = l´ım

2 (λe1 ) (λe2 )2 2λ3 e1 e22 2 4 (λe1 ) + (λe2 ) λ2 (e21 + λ2 e42 ) = l´ım Dbe f (0, 0) = l´ım λ→0 λ→0 λ λ Por lo tanto 2e2 Dbe f (0, 0) = l´ım 2 ,este l´ımite existe si y solo si e1 6= 0 λ→0 e1 Problema 2   x (x + y) , si (x, y) 6= (0, 0) . Determine, si existe, Sea f (x, y) = x2 + y 2  0 , si (x, y) = (0, 0) la derivada direccional de f en P0 = (0, 0). Soluci´ on. Sea eb = (e1 , e2 ) un versor en una direcci´on cualquiera de IR2 tal que

f ((0, 0) + λ (e1 , e2 )) − f (0, 0) f (λe1 , λe2 ) − f (0, 0) = l´ım λ→0 λ→0 λ λ e1 (e1 + e2 ) Dbe f (0, 0) = l´ım 2 λ→0 λ (e2 1 + e2 ) Dbe f (0, 0) = l´ım

Este l´ımite existe si y solo si e1 (e1 + e2 ) = 0 =⇒ e1 = 0 o´ (e1 + e2 ) = 0 i) Si eb = (0, e2 ) = 0, Dbe f (0, 0) = 0

ii) Si eb = (e1 , −e1 ) = 0, Dbe f (0, 0) = 0. 258

Problema 3 Hallar la derivada direccional de f (x, y, z) = x2 yz 3 en el punto P0 = − (1, 1, −1) en la direcci´on de la tangente a la trayectoria : → r (t) = −t (e , 1 + 2sen (t) , t − cos (t)) . Soluci´ on.

Como f (x, y, z) = x2 yz 3 es una funci´on diferenciable en R3 ,entonces Dbt f (P0 ) = ∇f (P0 ) · b t, donde ∇f (P ) = (2xyz 3 , x2 z 3 , 3x2 yz 2 )

El punto P0 que corresponde a t = t0 es: → − r (t0 ) = (e−t0 , 1 + 2sen (t0 ) , t0 − cos (t0 )) = (1, 1, −1) =⇒ e−t0 = 1 As´ı, t0 = ln (1) = 0

El vector tangente a la curva es: ′ ′ → − − r (t) = (−e−t , 2 cos (t) , 1 + sen (t)) , entonces → r (0) = (−1, 2, 1) y el vector tangente unitario en ´esta direcci´on queda b t=

′ − → r (0) ′ − → k r (0)k

=

(−1,2,1) √ 6

Por tanto, la derivada direccional es Dbt f (P0 ) = (−2, −1, 3) ·

(−1,2,1) √ 6

=

√3 6

>0

El valor positivo, significa que f aumenta en ´esta direcci´on. Problema 4

Calcular derivada √ la √  direccional de f (x, y, z) = xy +xz −yz en el punto P0 = 2, − 2, 0 en direcci´on de la tangente a la curva determinada por las superficies x2 + y 2 + z 2 = 4, x + y + z = 0 Soluci´ on. Como f (x, y, z) = xy + xz − yz R3 ,entonces

es una funci´on diferenciable en

Dbt f (P0 ) = ∇f (P0 ) · b t, donde ∇f (P ) = (y + z, x − z, x − y) =⇒  √ √ √  ∇f (P0 ) = − 2, 2, 2 2 259

A´ un falta calcular el vector b t=

′ − → r (P0 ) ′ − → r (P )

, que es tangente a la curva k 0 k determinada por las superficies dadas.  ′ − − Sea C dada por → r (x) = (x, y (x) , z (x)) =⇒ → r ′ (x) = 1, y (x) , z ′ (x) , ′

donde y (x) , z ′ (x) se calculan impl´ıcitamente a partir del sistema de ecuaciones por derivaci´on con respecto a x, en el entendido que y = y (x) , z = z (x) . En efecto: 2x + 2yy ′ + 2zz ′ = 0 ′ ′ 1+y +z =0

· 12

⇐⇒

yy ′ + zz ′ = −x ′ ′ y + z = −1

Resolviendo el u ´ltimo sistema obtenemos y −x z 1 −1 1 −x + z ′ ′ = , z (x) = y (x) = y y−z y z 1 1 1

−x −1 x−y = y−z z 1

Evaluando √ √ − 2 2 2 ′ √  ′ √  y 2 = √ =1yy 2 = √ = −2 − 2 − 2 Luego ′ → − √  (1, 1, −2) r (P0 ) → − ′ b √ =⇒ r 2 = (1, 1, −2) =⇒ t = → = ′ − k r (P0 )k 6 Por lo tanto, se tiene que √ √ √ −4 (1, 1, −2) √ Dbt f (P0 ) = (− 2, 2, 2 2) · =√ 6 6

3.4.6.

Valores extremos

Problema 1 Sea f : U ⊆ R2 → R definida en el abierto U , dada por f (x, y) = x4 − 2px2 − y 2 + 3 donde p es una constante entera. Calcular los valores extremos de la funci´on Soluci´ on Apliquemos la condici´on necesaria de punto cr´ıtico, calculando las derivadas parciales e igual´andolas a cero, luego fx = 4x3 − 4px = 4x(x2 − p); fy = −2y. 260

Tenemos que fy = −2y = 0 =⇒ y = 0, pero en la derivada parcial respecto de x hay que analizar el signo de p. √ i) Si p > 0, f x = 4x(x2 − p) = 0 =⇒ x = 0, ± p y los puntos cr´ıticos son √ √ P1 = (0, 0), P2 = ( p, 0) y P3 = (− p, 0). ii) Si p = 0, f x = 4x3 = 0 =⇒ x = 0, y el u ´nico punto cr´ıtico es P1 = (0, 0). iii) Si p < 0, f x = 4x(x2 − p) = 0 =⇒ x = 0, y el u ´nico punto cr´ıtico es P1 = (0, 0) Estudiemos el Hessiano para ver si los puntos obtenidos son m´aximos, m´ınimos o puntos silla. fxx = 12x2 − 4p, fxy = 0, fyx = 0, fyy = −2

Por tanto el determinante de la matriz Hessiana es 12x2 − 4p 0 H (x, y) = 0 −2



Estudiemos los puntos cr´ıticos seg´ un los valores del par´ametro p 1) Si p > 0: En este caso hay 3 −4p 0 H (0, 0) = 0 −2

puntos cr´ıticos. = 8p > 0 y fxx = −4p < 0

Entonces, el punto P1 = (0, 0) es un M´aximo Relativo. 8p 0 √ = −16p < 0 H (± p, 0) = 0 −2

√ √ Por lo tanto, los puntos P2 = ( p, 0) y P3 = (− p, 0) son Puntos Silla.

2) Si p < 0: En este caso en P1 = (0, 0) hay un punto cr´ıtico. H (0, 0) = −4p 0 0 −2 = 8p < 0, luego en P1 = (0, 0) hay un Punto Silla. 3) Si p = 0: En este caso hay un u ´nico punto cr´ıtico en el origen, pero al calcular el determinante del Hessiano, obtenemos que H(0, 0) = 0, luego no se tiene informaci´on. 261

Sin embargo, podemos comprobar la naturaleza del punto aplicando la definici´on de extremo local en las cercan´ıas del origen, siguiendo dos trayectorias distintas, para la funci´on: f (x, y) = x4 − y 2 + 3. Entonces f (x, 0) − f (0, 0) = x4 > 0 para todo (x, 0) ∈ V (0, 0) f (0, y) − f (0, 0) = −y 2 < 0 para todo (0, y) ∈ V (0, 0). Por lo tanto, P1 = (0, 0) es un Punto de Silla

Problema 2 Encuentre los valores extremos de la f (x, y) = x2 + y 2 − 21 x4 . Soluci´ on. Derivando parcialmente con respecto a x e y tenemos: fx (x, y) = 2x − 2x3 fy (x, y) = 2y Esta claro que fx y fy son funciones continuas en R2 Aplicando la condici´on necesaria para los puntos cr´ıticos de f tenemos: 2x − 2x3 = 0;

2y = 0.

Al resolver el sistema obtenemos tres puntos cr´ıticos. P0 = (0, 0) , P1 = (1, 0) ,

P2 = (−1, 0)

Determinemos el Hessiano H (x, y) : fxx (x, y) fxy (x, y) 2 − 6x2 0 = 4 − 12x2 H (x, y) = = fyx (x, y) fyy (x, y) 0 2

Evaluemos el Hessiano H (x, y) en cada uno de los puntos:

i) Para P0 = (0, 0) =⇒ H (0, 0) = 4 > 0 y fxx (0, 0) = 2 > 0 Entonces en P0 hay un m´ınimo relativo f (0, 0) = 0. ii) Para P1 = (1, 0) =⇒ H (1, 0) = −8 < 0 . Entonces, en P1 hay punto silla de f. iii) Para P2 = (−1, 0) =⇒ H (−1, 0) = −8 < 0 . As´ı, en P2 tambien hay un punto silla de f. 262

Problema 3 Encuentre los valores extremos de la f (x, y) = x2 + y 2 − xy + x + y en el dominio D = {(x, y) ∈ R2 / x ≤ 0, y ≤ 0, x + y ≥ −3} .

Soluci´ on.

En primer lugar, determinemos los valores extremos en el conjunto abierto: D∗ = {(x, y) ∈ R2 / x < 0, y < 0, x + y > −3} .

Derivando parcialmente con respecto a x e y tenemos: fx (x, y) = 2x − y + 1

fy (x, y) = 2y − x + 1

Observe que fx y fy son continuas en R2 Aplicando la condici´on necesaria para los puntos cr´ıticos de f tenemos el sistema: 2x − y = −1;

−x + 2y = −1.

Al resolver este sistema obtenemos un u ´nico punto cr´ıtico P0 = (−1, −1) ∈ D∗

Determinemos el Hessiano H (x, y) : fxx (x, y) fxy (x, y) 2 −1 = 3 ∀ (x, y) ∈ D∗ : = H (x, y) = fyx (x, y) fyy (x, y) −1 2

As´ı, P0 = (−1, −1) =⇒ H (−1, −1) = 3 > 0 y fxx (−1, −1) = 3 > 0 Se concluye, que en P0 hay un m´ınimo relativo f (−1, −1) = −1. En segundo lugar, estudiemos la condici´on que se presenta en la frontera de D. a) Si y = 0,

f (x, 0) = x2 + x con x ∈ [−3, 0]

Determinemos los puntos cr´ıticos en este borde f (x) = 2x + 1 = 0 =⇒ x = − 21 ′

 Luego, hay un punto crit´ıco en P1 = − 12 , 0 ∈ D.

 ′′ Como f (x) = 2 > 0, ∀x ∈ [−3, 0] , entonces en P1 = − 21 , 0 hay  un m´ınimo f − 21 , 0 = − 14 . b) Si x = 0,

f (0, y) = y 2 + y con y ∈ [−3, 0]

Determinemos los puntos cr´ıticos en este borde 263

f (y) = 2y + 1 = 0 =⇒ y = − 12 ′

 Luego, se tiene un punto crit´ıco en P2 = 0, − 21 ∈ D.

 ′′ Como f (y) = 2 > 0, ∀y ∈ [−3, 0] ,entonces en P2 = 0, − 21 hay  un m´ınimo f 0, − 12 = − 14 . c) Si x+ y = −3,

f (x, −x − 3) = 3x2 + 9x + 6 con x ∈ [−3, 0]

Determinemos los puntos cr´ıticos en este borde

f (x) = 6x + 9 = 0 =⇒ x = − 23 =⇒ y = − 23  Luego, hay un punto crit´ıco en P2 = − 32 , − 23 ∈ D. ′

′′

Como f (x, −x − 3) = 6 > 0, ∀x ∈ [−3, 0] ,entonces P2 = − 32 , − 23  en hay un m´ınimo f − 23 , − 23 = − 43 .



3.4.7. Multimplicadores de Lagrange para extremos restringidos Problema 1 Obtener los extremos de la funci´on f (x, y) = x3 + 2xy + y 2 que se encuentren en la recta x + y = 0. Soluci´on En primer lugar consideremos la funci´on de Lagrange y apliquemos la condici´on necesaria de punto cr´ıtico: F (x, y, λ) = x3 + 2xy + y 2 + λ(x + y) Fx = 3x2 + 2y + λ = 0 F y = 2x + 2y + λ = 0 Fz = x + y = 0

(1) (2) (3)

De la (3) ecuaci´on obtenemos: y = −x que sustituida en la segunda:

2x − 2x + λ = 0 =⇒ λ = 0

Sustituido este valor en la ecuaci´on (1), queda: 3x2 − 2x = 0, x = 0, x = 2/3   2 2 ,− Entonces los puntos cr´ıticos del lagrangiano son. P1 (0, 0), P2 3 3 . Clasificaremos los puntos con el determinante del Hessiano Limitado: 264

0 1 1 H (x, y) = 1 6x 2 1 2 2



evalu´ando en los puntos cr´ıticos obtenidos anteriormente. 0 1 1 d2 f < 0 y en Consideremos H (0, 0) = 1 0 2 = 2 > 0, entonces dx2 1 2 2 P1 (0, 0) hay un m´aximo local condicionado de f, cuyo valor es f (0, 0) = 0.  0 1 1  d2 f 2 2 = 1 4 2 = −2 > 0, entonces 0

Por lo tanto, en el punto P0 =



a b √ ,√ 2 2



un m´ınimo de f

Problema 3 Se desea construir una tolva para un silo, que tenga una capacidad de 100 m3 y forma de cono circular recto de 2m de radio, coronado por un cilindro circular recto, empleando un m´ınimo de material para la superficie. Calcular las alturas x del cilindro e y del cono para tal objeto. Soluci´ on: Sea la funci´on superficie definida por f (x, y) = 2π

p 4 + y 2 +4πx

Con la condici´on que el volumen sea g (x, y) = 34 πy + 4πx − 100 = 0

Entonces formemos la funci´on: p L (x, y, λ) = f (x, y) = 2π 4 + y 2 +4πx + λ Lx (x, y, λ) =

4π + 4πλ = 0 4 4πy + πλ = 0 Ly (x, y, λ) = p 2 3 2 4+y Lλ (x, y, λ) = 43 πy + 4πx − 100 = 0

4 πy 3

+ 4πx − 100

=⇒ λ = −1 1 y = =⇒ p 2 3 2 4+y



4 =⇒ y = √ 5 Sustituyendo en la restricci´on se tiene 9y 2 = 4 (4 + y 2 )

=⇒ 5y 2 = 16

100 − 3√4 5 4π En consecuencia, se tiene un u ´nico punto cr´ıtico en   100 4 4 P0 = −√ ,√ 4π 5 5 La condici´on de m´ınimo de f se estable mediante la segunda derivada p 4πyy ′ ′ 2 + 4π f (x, y(x)) = 2π 4 + y + 4πx =⇒ f (x) = p 2 4 + y2 4 πy + 4πx − 100 = 0 =⇒ y ′ (x) = −3 3 4πx = 100 −

16 √ π 3 5

=⇒ x =

Por lo tanto, sustituyendo y ′ (x) , y derivando por segunda vez 267



6πy

f (x) = − p + 4π 4 + y2 ′′ f (P0 ) > 0

′′

f (x) = −6π

=⇒

(−3(4 + y 2 ) − y 2

(4 + y 2 )3/2 Valor m´ınimo

=⇒

!

As´ı el valor m´ınimo de la funci´on es:     q 100 20π 4 4 4 100 16 = 2π 4 + 5 +4π = 100 − √ f −√ ,√ −√ 4π 4π 5 5 5 3 5 Problema 4 Determine la distancia m´ınima y m´axima del origen a la curva de in7 tersecci´on del paraboloide z = − x2 − y 2 y el plano x + y + z = 2. 4 Soluci´on: En este caso es conveniente los valores extremos del cuadrado de la distancia con respecto al origen en vez de la distancia misma. Por lo tanto, se deben hallar los valores extremos de la funci´on: f (x, y, z) = x2 + y 2 + z 2 sujeta a las restriciones 7 + x2 + y 2 = 0 4 h (x, y, z) = x + y + z − 2 = 0 g (x, y, z) = z −

Para aplicar el m´etodo de los multiplicadores de Lagrange se define   7 2 2 2 2 2 F (x, y, z, λ1 , λ2 ) = x +y +z +λ1 z − + x + y +λ2 (x + y + z − 2) 4 Fx = 2 (1 + λ1 ) x + λ2 = 0 Fy = 2 (1 + λ1 ) y + 2λ2 = 0 Fz = 2z + λ1 + λ2 = 0 7 F λ1 = z − + x2 + y 2 = 0 4 F λ2 = x + y + z − 2 = 0

(1,0) (2,0) (3,0)

(4,0) (5,0)

1,0) − 2,0) : 2 (1 + λ1 ) (x − y) = 0 =⇒ λ1 = −1 o y = x 1 Si λ1 = −1 ,entonces de 1) λ2 = 0 y de 3) z = 2 268

1 Si z = , entonces de 4) y 5) se obtiene: 2x2 − 3x + 1 = 0 2 1 Resolviendo la ecuaci´on anterior, sus soluciones son: x1 = 1; x2 = 2   1 1 1 x1 = 1 =⇒ y1 = =⇒ 1, , es punto cr´ıtico. 2 2 2   1 1 1 x2 = es punto cr´ıtico. =⇒ y2 = 1 =⇒ , 1, 2 2 2 Por otra parte:4) − 5) =⇒ x2 + y 2 − x − y +

1 =0 4

√ 2 ± 2 1 Si y = x =⇒ 2x2 − 2x + = 0, resolviendo la ecuaci´on x = 4 4 √ √ √ √ ! √ 2+ 2 2+ 2 4−2 2 4−2 2 2+ 2 =⇒ z = =⇒ , , y=x= 4 4 4 4 4 es punto cr´ıtico de f. √ √ 2− 2 4+2 2 y=x= =⇒ z = =⇒ 4 4

√ √ √ ! 2− 2 2− 2 4+2 2 , , 4 4 4

es punto cr´ıtico de f. As´ı

√ √ ! √ √  1 2± 2 2± 2 4∓2 2 9+2 2 , , = fm´ax 4 4 4 4   1 3 1 fm´ın = , 1, 2 2 2 Como la curva intersecci´on del paraboloide y el plano es una curva cerrada, la distancia m´ınima y la distancia m´axima al origen son rer q √  3 1 9 + 2 2 .No necesitamos m´as pruebas por y spectivamente 2 2 las caracteristicas geom´etricas del problema. Problema 5 Demuestre que las distancias m´axima y m´ınima desde el origen a la x2 y 2 z 2 + + = 1, z = x + y. curva de intersecci´on definida por 4 5 25 Soluci´ on: 269

Debenos encontrar los valores extremos de la funci´on f (x, y, z) = x2 + y 2 + z 2 sujeta a las restriciones x2 y 2 z 2 + + −1=0 4 5 25 h (x, y, z) = x + y − z = 0 g (x, y, z) =

Para aplicar el m´etodo de los multiplicadores de Lagrange se define  2  x y2 z2 2 2 2 F (x, y, z, λ1 , λ2 ) = x +y +z +λ1 + + − 1 +λ2 (x + y − z) 4 5 25 Aplicando la condici´on necesaria de punto cr´ıtico   λ1 Fx = 2 1 + x + λ2 = 0 (1,0) 4  λ1 Fy = 2 1 + y + λ2 = 0 (2,0) 5 λ1 Fz = 2(1 + + λ2 = 0 (3,0) 5 x2 y 2 z 2 + + −1=0 (4,0) F λ1 = 4 5 25 F λ2 = x + y − z = 0 (5,0)

Despejando de estas ecuaciones x, y, z se tiene 2λ2 5λ2 25λ2 x=− ; y=− ; z=− ; 6,0) λ1 + 4 2λ1 + 10 2λ1 + 50 Al dividir 5,0 por λ2 6= 0 (lo cual est´a justificado porque de otro modo de 1,0, 2,0 y 3,0, se tendr´ıa x = y = z = 0). 5 25 2 + + = 0. Multiplicando por 2 (λ1 + 4) (2λ1 + 10) (2λ1 + 50) λ1 + 4 2λ1 + 10 2λ1 + 50 y simplificando da 17λ21 + 245λ1 + 750 = 0 =⇒ (λ1 + 10)(17λ1 + 75) = 0 75 de donde: λ1 = −10, λ1 = − 17 λ2 λ2 5λ2 Caso i) Si λ1 = −10, entonces de 6,0 : x = ; y = ; z = . 3 2 6 180 λ22 λ22 5λ22 + + − 1 = 0 =⇒ λ22 = =⇒ Sutituyendo en 4.0 da: 36 20 66 19 270

r

5 19 Por lo tanto, se tienen dos puntos cr´ıticos. r r ! r r ! r r 5 5 5 5 5 5 ,3 ,5 .−3 , −5 y P2 = −2 P1 = 2 19 19 19 19 19 19 ! r r r 5 5 5 = 10 , ±3 , ±5 Evaluando en la funci´on se tiene f ±2 19 19 19 λ2 = ±6

75 34λ2 17λ2 Caso ii) Si λ1 = − , entonces de 6,0 : x = ; y =− ;z = 17 7 4 17λ2 . 28 λ2 λ2 5λ2 (140)2 Sutituyendo en 4.0 da: 2 + 2 + 2 −1 = 0 =⇒ λ22 = =⇒ 36 20 66 (17)2 (646) √ λ2 = ±14017 646 Por lo tanto, se tienen otros dos puntos cr´ıticos m´as.     5 40 35 35 5 40 √ y P2 = − √ P1 = √ , −√ ,√ , −√ 646 646 646 646 646 646   40 75 35 5 Evaluando en la funci´on se tiene f ± √ = , ∓√ , ±√ 17 646 646 646 75 Asi el valor m´aximo buscado es 10 y el valor m´ınimo es 17 Problema 6 Se desea construir un silo, que tenga una capacidad de V0 con forma de cilindro circular recto de altura h y radio basal r . Calcular la altura h del cilindro y radio basal r de manera que la superficie total sea m´ınima. Soluci´ on: Sea la funci´on superficie definida por f (r, h) = 2πr2 +2πrh Con la condici´on que el volumen sea g (x, y) = πr2 h − V0 = 0 Entonces formemos la funci´on: L (r, h, λ) = 2πr2 + 2πrh + λ (πr2 h − V0 ) 271

Lr (r, h, λ) = 4πr + 2πh + 2λπrh = 0 1,0) Lh (r, h, λ) = 2πh + λπr2 = 0 2,0) Lλ (r, h, λ) = πr2 h − V0 = 0 3,0) 2 De 2,0) se tiene: λ = − y sustituyendo este valor en 1,0) obtenemos r h = 2r r r V0 3 V0 Si h = 2r, entonces de 3.0) r = ;h = 2 3 2π 2π En consecuencia, se tiene un u ´nico punto cr´ıtico en r ! r V0 3 V0 P0 = 3 ,2 2π 2π La condici´on de m´ınimo de f se establece mediante la segunda derivada f (r, h(r)) = 6πr2



=⇒ f (r) = 12πr ′′ =⇒ f (r) = 12π > 0

Por lo tanto, se tiene un valor m´ınimo de f si h = 2r As´ı el valor m´ınimo de la superficie es: r ! r  2/3 V V0 0 3 V0 3 ,2 = 6π f 2π 2π 2π Problema 7: Determinar los extremos absolutos de la funci´on f (x, y) = y 3 + x2 y + 2x2 + 2y 2 − 4y − 8 en el conjunto D = {(x, y) ∈ IR2 /x2 + y 2 ≤ 1} . Soluci´ on:

En primer lugar estudiemos los puntos del interior de D, para ver si existen m´aximos o m´ınimos locales. La condicion necesaria, de los puntos interiores candidatos a extremos, es ∂f = 2x(y + 2) = 0 ∂x ∇f (x, y) = (0, 0) =⇒ ∂f = 3y 2 + x2 + 4y − 4 = 0 ∂y i) La primera ecuaci´on implica que x = 0 ´o y = −2. Si y = −2, la segunda ecuaci´on implica que x = 0, luego se tiene un punto cr´ıtico en P0 = (0, −2) , sin embargo , P0 ∈ / D. 272

ii) Si x = 0, la segunda ecuaci´on es 3y 2 + 4y − 4 = 0 =⇒ y = −2, 2 y= . 3   2 4 Las coordenadas del punto P1 = 0, verifican < 1,entonces P1 ∈ 3 9 8 8 256 8 + − −8= = −9, 48. D. Adem´as f (P1 ) = 27 9 3 27 En segundo lugar, estudiemos los puntos de la frontera de D usando la funci´on f (x, y) = y 3 + x2 y + 2x2 + 2y 2 − 4y − 8 bajo la restrici´on g (x, y) = x2 + y 2 − 1 = 0. Usemos el m´etodo de los multiplicadores de Lagrange.

Sea L (x, y, λ) = y 3 + x2 y + 2x2 + 2y 2 − 4y − 8 + λ(x2 + y 2 − 1),

y obtenemos: ∂L = 2x(y + 2) + λ2x = 0 ∂x ∂L = 3y 2 + x2 + 4y − 4 + λ2y = 0 ∂y ∂L = x2 + y 2 − 1 = 0 ∂λ De la ecuaci´on 1,0 se tiene que x = 0 o´

(1,0) (2,0) (3,0) (y + 2) + λ = 0

a) Si x = 0,en 3.0 se tiene y 2 − 1 = 0 =⇒ y = ±1.Luego se tienen otros dos puntos cr´ıticos P2 = (0, 1) y P2 = (0, −1) que satisfacen las ecuaciones 1,0 y 3,0.

Para comprobar que tambi´en satisfacen la ecuaci´on 2,0 , sustituyamos el ella 3 P1 = (0, 1) =⇒ λ = ∈ IR 2 5 P2 = (0, −1) =⇒ λ = ∈ IR. 2 Si evaluamos las funci´on en los puntos encontrados obtenemos: f (P2 ) = 1 + 2 − 4 − 8 = −9

f (P3 ) = −1 + 2 + 4 − 8 = −3

b) Si (y + 2) + λ = 0 ⇐⇒ λ = −(y + 2), en 2,0 se tiene 3y 2 + x2 + 4y − 4 + 2y(y + 2) = 0 =⇒ x2 + y 2 = 4, resultado que contradice la ecuaci´on 3,0, x2 + y 2 = 1.

Luego, esta condici´on no produce un punto cr´ıtico. 273

Por lo tanto, comparando los valores de la funci´on en los tres puntos encontrados, podemos inferir que el m´aximo absoluto se alcanza en P3 y que el m´ınimo absoluto se alcanza en P1 Problema 8: Determine las dimensiones de una caja rectangular, sin tapa superior,que ha de tener un volumen dado V0 ,de manera que su superficie sea m´ınima. Soluci´ on: Sea la funci´on superficie definida por f (x, y, z) = xy + 2xz + 2yz Con la condicion que el volumen sea g (x, y, z) = xyz − V0 = 0

Entonces formemos la funci´on:

L (x, y, z, λ) = f (x, y) = xy + 2xz + 2yz + λ (xyz − V0 ) Lx (x, y, z, λ) = y + 2z + λyz = 0 Ly (x, y, z, λ) = x + 2z + λxz = 0 Lz (x, y, z, λ) = 2x + 2y + λxy = 0 Lλ (x, y, z, λ) = xyz − V0 = 0

(1,0) (2,0) (3,0) (4,0)

2z , sutituyendo Despejando y y x de 1,0 y 2,0 se tiene 5.0) y = x = − 1+λz en 3,0) produce: 8z + − 1+λz

4λz 2 (1+λz)2

= 0 =⇒ z =

1 λ

6,0)

Reemplazando en 6,0) en 5,0) se tiene y = x = − λ4 . Sustituyendo en 4.0  1/3   1 4 4 − λ − λ λ = V0 =⇒ λ = V160

Por lo tanto, se tiene un u ´nico punto cr´ıtico de f en    1/3  1/3 1/3 . , 4 V160 , V160 P0 = 4 V160

Examinemos la naturaleza del punto cr´ıtico usando el Hessiano limitado: ∂g ∂g ∂g 0 ∂x ∂y ∂z ∂g ∂ 2 L ∂ 2 L ∂ 2 L ∂x ∂x2 ∂x∂y ∂x∂z H (x, y, z) = ∂g ∂ 2 L ∂ 2 L ∂ 2 L =⇒ ∂y ∂x∂y ∂y2 ∂y∂z ∂g ∂ 2 L ∂ 2 L ∂ 2 L 2 ∂z

∂x∂z

∂y∂z

∂z

274

0 yz xz xy yz 0 1 + λz 2 + λy xz 1 + λz 0 2 + λx xy 2 + λy 2 + λx 0  1/3 1/3 V0 1/3  Evaluemos el Hessiano H 4 V160 , 4 V160 , 16 2/3 2/3 2/3 4 V160 16 V160 0 4 V160 2/3 V 0 4 0 2 6 16 H=  2/3 V0 2 0 6 4 16 2/3 V 0 16 16 6 6 0 H (x, y, z) =

Usando propiedades de determinantes, obtenemos que el valor de Hessiano es:  V0 4/3 H = 16 16

0 1 1 4

1 0 2 6

1 2 0 6

4 6 6 0 4



H = −2048,0 × (0,062 5V0 ) 3 < 0 0 1 1 Adem´as: ∆3 = 1 0 2 = 4 > 0 1 2 0

Entonces  lafunci´on f tendr´a un m´aximo condicionado en el punto 1/3 1/3 1/3 , 4 V160 , V160 P0 = 4 V160

3.4.8.

Aplicaci´ on al c´ alculo de errores

Problema 1: El periodo T de un p´endulo simple depende de la longitud l y de la aceleraci´on de gravedad g del lugar y est´a dado por: r l T = 2π . Hallar a a) el error absoluto y b) el error relativo , al g calcular T con l = 0, 6 m y g = 10m/s2 si los valores verdaderos eran l = 58, 5cm y g = 9, 8m/s2 . Soluci´ on: a) Sea T = 2π

r

l . el per´ıodo de un p´endulo simple. g 275

El error absoluto de T es ∆T, que en este caso es aproximadamente dT. as´ı se tiene: ∂T π ∂T dl + dg = √ dl − π El error absoluto de T = dT = ∂l ∂g lg Error de l = ∆l = dl = (0, 6 − 0, 585) m = 0, 015m

r

l dg g3

Error de g = ∆g = dg = (10 − 9, 8) m/s2 = 0, 2m/s2 r π 0, 6 El error absoluto de T = dT = √ (0, 2) (0, 015) − π 1000 0, 6x10 r   dT l π 1 √ dl − π b) El error relativo de T = = r dg T g3 lg l 2π g  1 1 dl − dg . El error relativo de T = 2l 2g r   π 1 dT l √ dl − π dg = r El error relativo de T = T g3 lg l 2π g 

3.5.

Ejercicios Propuestos

3.5.1.

L´ımites

Problema 1 Determinar si existen los siguientes limites, y si existen, calcular su valor a)

l´ım

(x,y)→(0,0)

ln xseny ysenx

x2 − y 2 (x,y)→(0,0) x2 + y 2 Soluci´ on: b)

l´ım

= 0, b) El l´ımite de la funci´on f no existe a) l´ım ln xseny ysenx (x,y)→0

276

Problema 2 En los siguientes casos, usar coordenadas polares para concluir que el l´ımite de f (x, y) cuando (x, y) tiende a (0, 0) existe y vale cero. a)

y3 (x,y)→(0,0) x2 + y 2

b)

x3 y 4 (x,y)→(0,0) x4 + y 4

l´ım

l´ım

3.5.2.

Diferenciabilidad, continuidad

Problema 1 Dada la funci´on f (x, y) =

 

xysen

1 si y

y 6= 0

, probar que es 0 ∀ (x, 0) diferenciable en el punto P0 = (0, 0) ,¿es continua en este punto? Soluci´ on:



La funci´on f es diferenciable en P0 = (0, 0) , implica que es continua en (0, 0) Problema 2   p |x| y si (x, y) 6= (0, 0) . Dada la funci´on f (x, y) = x2 + y 2  0 si (x, y) = (0, 0)

a) Estudiar su continuidad en IR2

b) Estudiar la existencia de derivadas parciales en IR2 y determinarlas, si es que existen. c) Estudiar su diferenciabilidad en IR2 Soluci´ on: a) La funci´on f es en continua en IR2 b) Existen derivadas parciales en ∂f ∂f (0, a) , (0, a) = 0, IR2 − {(o, a) /a 6= 0} ,No existe ∂x ∂y 277

∂f ∂f (0, 0) = 0, (0, 0) = 0 ∂x ∂y c) La funci´on f no es diferenciable en P0 = (0, 0) ,puesto que |∆f − df | √ 6= 0 (h,k)→(0,0) h2 + k 2 l´ım

Problema 3 Verificar que f (x, y) =

p |xy| es continua y no diferenciable en (0, 0) .

Problema 4 Dada la funci´on f (x, y) =

(

4

4

arctg xx2 +y si (x, y) 6= (0, 0) +y 2 a si (x, y) = (0, 0)

a) Determinar el valor de a para que la funci´on sea continua en el origen. b) Para este valor de a calcular fx (0, 0), fy (0, 0) c) Hallar la derivada direccional Dub f (1, 0), siendo u b un vector unitario o que la forma un ´angulo de 60 con la parte positiva del eje OX.

Soluci´ on:

a) f(x,y) es continua en (0,0) si a= f(0,0) b) fx (0,0)=0 , fy (0,0)=0 c) Dub f (1, 0) =

3.5.3.

1 2

Derivadas parciales

Problema 1 Sea f (x, y) =

  

xy

x2 − y 2 si (x, y) 6= (0, 0) x2 + y 2 0 si (x, y) = (0, 0)

a) Deducir que

∂f ∂f (0, y) = −y ∀y; (x, 0) = x ∀x ∂x ∂y

b) Verificar que

∂ 2f ∂ 2f (0, 0) 6= (0, 0) ∂y∂x ∂x∂y 278

Problema 2 Dada la funci´on f (x, y) =



x + y si x = 0 ´o y = 0 1 si x = 6 0, y 6= 0

∂f ∂f (0, 0) = 1 ; (0, 0) = 1 ∂x ∂y b) Muestre que la funci´on f no es continua en (0, 0)

a) Verifique que

Problema 3 Sea u =

xy , muestre que u satisface la ecuaci´on: x+y x2

2 ∂ 2u ∂ 2u 2∂ u + 2xy =0 + y ∂x2 ∂y∂x ∂y 2

Problema 4 Sea u = p

1 x2

+ y2 + z2

, probar que u satisface la ecuaci´on: ∂ 2u ∂ 2u ∂ 2u + + =0 ∂x2 ∂y 2 ∂z 2

Problema 5 Enuncie las condiciones del teorema de Schwarz de igualdad de las derivadas cruzadas. Problema 6 Justifique si es cierto que una funci´on f : IR3 → IR diferenciable en IR3 tiene derivadas parciales. Soluci´on − − Si una funci´on f es diferenciable en → x o y la diferencial es Df (→ x o) , → − entonces para todo vector unitario vb 6= 0 existe D f ( x ) que verifica v b

o

− − Dvbf (→ x o ) = Df (→ x o ) · vb. En particular, esto sucede para los vectores (1, 0, 0) , (0, 1, 0) , (0, 0, 1) , por lo que existen las derivadas parciales

279

Problema 7 Suponga que z es funci´on de las variables x e y , que satisface la ecuaci´on que se da en cada caso . Encontrar las derivadas parciales de primer ∂z ∂z orden y ∂x ∂y a) x3 + y 3 + z 3 + senxz + cos yz = 15 b) ez + x2 ln z + y = 0. Soluci´ on: ∂z (3x2 + z cos xz) ∂z (3y 2 − zsenxz) a) =− 2 , =− 2 ∂x 3z + x cos xz − ysenyz ∂y 3z + x cos xz − ysenyz 2xz ln z ∂z z ∂z , =− z =− z b) 2 ∂x ze + x ∂y ze + x2 Problema 8 Suponga que existen funciones u y v , que satisfacen las siguientes ecuaciones. 

u cos v = x + 1 u sin y = x + y

∂u ∂u ∂v ∂v , , , ∂x ∂y ∂x ∂y Soluci´ on: ∂u ∂u ∂v cos v − senv ∂v cos v = cos v + senv , = senv, = , = ∂x ∂y ∂x u ∂y u

Calcular

3.5.4.

Derivadas Direccionales

Problema 1 Calcule la derivada direccional de la funci´on dada en la direcci´on del vector indicado. a) f (x, y, z) = 2x2 − y 2 − z 2 , en (1, 2, 2) hacia (4, 5, 0) .

b) f (x, y, z) = 2x2 − 8xy + z 2 , en el punto (4, 4, 1) en la direcci´on de la 280

normal exterior a la superficie x2 + y 2 + z = 17 Soluci´ on: 446 8 a) Dvbf (1, 2, 2) = √ , b) Dvbf (4, 4, 1) = − √ 22 129 Problema 2

Determine la ecuaci´on del plano tangente al paraboloide x2 +y 2 +z−1 =   1 1 0 y que pasa por los puntos 1, 0, 2 y 0, 1, 2 . Soluci´ on:

La ecuaci´on de plano tangente es: x + y + z =

3 2

Problema 3 Sea S una superficie dada por la ecuaci´on x3 + y 3 + z 3 − a3 = 0, con a constante, y Π un plano tangente en P0 = (xo , yo, zo ) ∈ S que intersecta los ejes coordenados X,Y,Z en α, β, γ respectivamente. Probar que α−3/2 + β −3/2 + γ −3/2 = a−3/2 es constante. Problema 4 Determinar el plano tangente a la superficie S √

x+



y+



z=



a, (a > 0)

en el punto P0 = (xo , yo, zo ) ∈ S y demostrar que este plano corta los ejes coordenados en segmentos cuya suma de longitudes es constante. Soluci´ on: x − xo y − yo z − z o + √ + √ =0 √ xo yo zo √ √ √ √ l = a( xo + yo + zo ) = a, (a > 0) 281

Problema 5 Muestre que si el punto P0 = (xo , yo, zo ) pertenece al elipsoide de ecuaci´on: x2 y2 z2 π: 2 + 2 + 2 = 1 con (α < c2 < b2 < a2 ) a −α b −α c −α y al manto del hiperboloide de ecuaci´on: y2 z2 x2 + 2 + 2 = 1 con (c2 < β < b2 < a2 ) θ: 2 a −β b −β c −β Entonces las superficies π y θ se cortan ortogonalmente en P0 = (xo , yo, zo ) . Regla de la cadena Problema 1 Sean f : R2 → R y g : R → R, con f (x, g (x)) = x sin (g (x)) . ′ Sabiendo que g (1) = 0 y que f (1, 0) = 1, calcular g ′ (1) . Soluci´ on: El valor de la derivada es g ′ (1) = 1 Problema 2 Sea f : D ⊂ IR2 → IR grado p, es

una funci´on diferenciable homog´enea de

decir que verifica la condici´on f (tx, ty) = tp f (x, y) ∀t > 0, ∀ (x, y) ∈ IR2 . ∂f ∂f +y = pf. a) Demuestre la ecuaci´on x ∂x ∂y b) Pruebe que la siguientes funciones son homog´eneas f (x, y) = xy, f (x, y) = x2 + 3xy + y 2 y luego verifique la ecuaci´on anterior. Problema 3 Sea g(t) = f (x (t) , y (t)) de clase C2 , deduzca que:

2

2

g ′′ (t) = fxx (x, y) (x′ ) +2fxy (x, y) x′ y ′ +fyy (x, y) (y ′ ) +fx (x, y) x′′ +fy (x, y) y ′′ 282

Problema 4 Sea f = f (x, y) de clase C2 con fx (0, 1) = 2, fy (0, 1) = 1; fxx (0, 1) = 0, fxy (0, 1) = −1, fyy (0, 1) = 1.Si h(t) = f (t2 , 1 + t3 ) . ′ − Calcule la derivada compuesta (f ◦ → r ) (t) y evalue h´(0) = 4

Soluci´ on:

h (t) = fxx (x, y) (x′ )2 + 2fxy (x, y) x′ y ′ + fyy (x, y) (y ′ )2 + fx (x, y) x′′ + fy (x, y) y ′′ ′′

h′′ (0) = 4 Derivaci´on impl´ıcita Sea z = f (x, y) definida por z = u + vdonde u = u(x, y) y v = v(x, y) son funciones definidas de manera impl´ıcita por las ecuaciones F = u + eu+v − x = 0

G = v + eu−v − y = 0

Si u = v = 0 entonces x = y = 1: Calcular zx (1, 1).

Soluci´ on: zx (1, 1) = 1 Problema 5 Verifique que la funci´on f (x, y) = g 2x



x y2



satisface la ecuaci´on:

∂f ∂f +y =0 ∂x ∂y

Problema 6   xy Sea g , x2 + y 2 = 0 una ecuaci´on que define a z como una funci´on z de x e y. Verifique que si gx ; gy y gz existen y son continuas en toda la regi´on en la que gz 6= 0; entonces 283

z(x2 − y 2 ) yzx − xzy = xy Problema 7 Si F (xz, yz) = 0 define a z como funci´on impl´ıcita de x e y y adem´as cumple con las condiciones del teorema de la funci´on impl´ıcita en cada punto de una regi´on R; entoncesverifique que, en R; se satisface la ecuaci´on xzx + yzy = −z

3.5.5.

Puntos cr´ıticos m´ aximos y m´ınimos

Problema 1 Sea f : U ⊆ R2 → R definida en el abierto U , dada por:

f (x, y) = x4 + y 4 + 4axy + 8a4 , (a ∈ R). Calcular los valores extremos de la funci´on. Soluci´ on: Si a = 0, entonces, el u ´nico punto cr´ıtico es P0 = (0, 0) (m´ınimo) . Si a >√ 0, tenemos tres puntos √ √ √ cr´ıtico es P0 = (0, 0) (punto silla), P1 = ( a, − a), P2 = (− a, a) (m´ınimos).

Si a 0 y ε > 0, podemos escoger n´ umeros ρo > 0 y εo > 0 tales que un punto situado en cualquier lugar a una distancia − menor que ρo de → r o ,despu´es de recibir inicialmente energ´ıa cin´etica en una cantidad menor que εo , permanecer´a para siempre a una distancia − de → r o menor que ρ y poseera energ´ıa cin´etica menor que ε.

− As´ı, si tenemos una posici´on de equilibrio, la estabilidad en → r o sig→ − nifica que una part´ıcula que se mueve lentamente cerca de r o siempre − permanecer´a cerca de → r o y se mantendr´a moviendose lentamente. − Ahora, si tenemos un punto de equilibrio inestable → r o , entonces → − → − → − → − − r (t) = r o resuelve la ecuaci´on de Newton F ( r (t)) = m→ r ” (t) , → − pero las soluciones cercanas pueden alejarse de r conforme trascurra o

el tiempo.

Proposici´ on 3.7.1. i) Los puntos cr´ıticos de un potencial son posiciones de equilibrio. − ii) En un campo conservativo, un punto → r o en el cual el potencial alcance un m´ınimo local estricto, es una posici´ on de equilibrio estable. 290

Demostraci´ on 1) La primera afirmaci´on es bastante obvia debido a la definici´on de → − − campo conservativo: F = −∇V, los puntos de equilibrios → r o son − exactamente los puntos cr´ıticos de V, en los cuales ∇V (→ r o) = 0 2) Para probar la afirmaci´on ii), haremos uso de la ley de conservaci´on de energ´ıa . Tenemos

1 1 − − − − m k→ r ′ (t)k + V (→ r (t)) = m k→ r ′ (0)k + V (→ r (0)) 2 2 − Escojamos un peque˜ na vecindad de → r , y asumamos que la part´ıcula o

tienen poca energ´ıa cin´etica. Conforme t crece, la part´ıcula se aleja − − − − de → r o sobre una trayectoria → r (t) y V (→ r (t)) crece pues V (→ r (0)) es un m´ınimo estricto , de modo que la energ´ıa cin´etica debe decrecer . Si la energ´ıa cinetica inicial es suficientemente peque˜ na, entonces, para − que la part´ıcula escape de la vecindad de → r o , fuera de la cual V ha crecido en una cantidad definida, la energia cinetica tendria que volverse negativa, lo cual es imposible. Asi, la particula no puede escapar de la vecindad.

Sea una part´ıcula en un campo de potencial V restringido a mantenerse sobre la superficie de nivel S dada por la ecuaci´on φ (x, y, z) = 0 → − − − con ∇φ 6= 0. Si en la ecuaci´on de Newton F (→ r (t)) = m→ r ” (t) (∗) , → − → − reemplazamos F con la componente de F paralela a S, aseguramos que la part´ıcula permanecer´a en S. Proposici´ on 3.7.2. i) Si en un punto P sobre la superficie S el potencial V |S tiene un valor extremo, entonces el punto P es una posici´ on de equilibrio sobre la superficie. ii) Si un punto P ∈ S es un m´ınimo local estricto del potencial V|S , entonces el punto P es una posicion de equilibrio estable.

Ejemplo Sea el campo gravitacional cerca de la superficie de la tier→ − ra; esto es, sea F = (0, 0, −mg) donde g es la aceleraci´on de gravedad . Determine la funci´on potencial gravitacional y ¿cu´ales son las posiciones de equilibrio, si una part´ıcula con masa m esta restrigida a la esfera g (x, y, z) = x2 + y 2 + z 2 − r2 = 0, (r > 0)?¿Cu´ales son estables?. Soluci´ on:

291

∂V = mg =⇒ V (x, y, z) = mgz ∂z Usando el m´etodo de los multiplicadores de Lagrange podemos localizar Tenemos que

Fz = −

los extremos posibles, tenemos que: L (x, y, z, λ) L (x, y, z, λ) Lx (x, y, z, λ) Ly (x, y, z, λ)

= = = =

V (x, y, z) + λg (x, y, z)  mgz + λ x2 + y 2 + z 2 − r2 2λx = 0 =⇒ λ 6= 0 y x = 0 (1,0) 2λy = 0 =⇒ λ 6= 0 y y = 0 (2,0) mg y z 6= 0 Lz (x, y, z, λ) = mg + 2λz = 0 =⇒ λ = − 2z Lλ (x, y, z, λ) = x2 + y 2 + z 2 − r2 = 0 ( 4,0)

(3,0)

Reemplazando (1,0), (2,0), (3,0) en ( 4,0) z 2 − r2 = 0 =⇒ z = ±r y λ = −

mg 2r

Luego, se deduce que los puntos P1 = (0, 0, r) y P2 = (0, 0, −r) son posiciones de equilibrio.

3.7.2.

Aplicaciones a la geometr´ıa

Ejemplo 1 Determine la distancia m´ınima desde el origen (0,0,0) a y2 z2 + = 1. la superficie S del elipsoide x2 + 4 9 Soluci´ on: Sabemos que la distancia pentre un punto P y el origen est´a dada por la funci´on d(x, y, z) = x2 + y 2 + z 2 . Sin embargo, por razones de simplicidad en los c´alculos , en lugar de la funci´on anterior vamos a considerar la funci´on f (x, y, z) = x2 + y 2 + z 2 en atenci´on a que f tendra un m´ınimo en un punto si y solo si d lo tiene. Se trata de obtener los extremos condicionados de la funci´on distancia y2 z2 + = 1. f (x, y, z) = x2 + y 2 + z 2 sujeta a la condici´on x2 + 4 9 Formemos la funci´on auxiliar de Lagrange y2 z2 + − 1) F (x, y, z, λ) = (x + y + z ) + λ(x + 4 9 2

2

292

2

2

y consideremos entonces el sistema. Fx (x, y, z, λ) = 2x + 2λx = 0 1 Fy (x, y, z, λ) = 2y + λy = 0 2 2 Fz (x, y, z, λ) = 2z + λz = 0 9 2 y z2 Fλ (x, y, z, λ) = x2 + + −1=0 4 9 De las tres primeras ecuaciones obtenemos 2x(1 + λ) = 0 =⇒ x = 0 ∨ λ = −1 1 y(2 + λ) = 0 =⇒ y = 0 ∨ λ = −4 2 1 2z(1 + λ) = 0 =⇒ z = 0 ∨ λ = −9 9 Sustituyendo y = z = 0 en la cuarta ecuaci´on produce x2 − 1 = 0 =⇒ x = ±1 . Reemplazando x = z = 0 en la cuarta ecuaci´on produce y2 − 1 = 0 =⇒ y = ±2 4 Sustituyendo x = y = 0 en la cuarta ecuaci´on produce z2 − 1 = 0 =⇒ z = ±3 9 Luego , se obtienen seis puntos cr´ıticos P0 = (1, 0, 0) , P1 = (−1, 0, 0) , P2 = (0, 2, 0) , P3 = (0, −2, 0) P4 = (0, 0, 3) , P5 = (0, 0, −3) .

Evaluando la funci´on en los puntos encontrados deber´a haber un m´aximo y un m´ınimo f (±1, 0, 0) = 1, f (0, ±2, 0) = 4, f (0, 0, ±3) = 9

Se tiene que el m´ınimo de f se encuentra en los puntos (±1, 0, 0) y es igual 1. y el m´aximo est´a localizado en los puntos (0, 0, ±3) y vale 9. 293

Ejemplo 2 Determine la distancia m´ınima y m´axima del origen a la 7 curva de interseccion del paraboloide x2 + y 2 + z − = 0 y el plano 4 x + y + z − 2 = 0.

Soluci´ on:

Igual que el ejemplo anterior resulta mas conveniente hallar los extremos del cuadrado de la distancia respecto del origen en vez de la p 2 funci´on distancia d (x, y, z) = x + y 2 + z 2 Por consiguiente, se trata de obtener los extremos condicionados de la funci´on distancia f (x, y, z) = x2 + y 2 + z 2 sujeta a las condiciones 7 g(x, y, z) = x2 + y 2 + z − = 0 y h(x, y, z) = x + y + z − 2 = 0 4

Formemos la funci´on auxiliar de Lagrange   7 2 2 2 2 2 F (x, y, z, λ1 , λ2 ) = (x +y +z )+λ1 x + y + z − +λ2 (x + y + z − 2) 4 y consideremos entonces el sistema. Fx (x, y, z, λ1 , λ2 ) = 2x(1 + λ1 ) + 2λ2 = 0 (1) Fy (x, y, z, λ1 , λ2 ) = 2y(1 + λ1 ) + 2λ2 = 0 (2) Fz (x, y, z, λ1 , λ2 ) = 2z + λ1 + λ2 = 0 (3) 7 Fλ1 (x, y, z, λ1 , λ2 ) = x2 + y 2 + z − = 0 (4) 4 Fλ2 (x, y, z, λ1 , λ2 ) = x + y + z − 2 = 0 (5) De las dos primeras ecuaciones se obtiene λ1 = −1 ´o y = x

Consideremos primero el caso λ1 = −1.A partir de (1) se obtiene λ2 = 1 0 . Sustituyendo estos valores en (3) z = . 2 Reemplazando z en (4) y (5), produce 5 = 0 4 3 = 0 x+y− 2

x2 + y 2 −

Resolviendo el sistema se obtienen los puntos cr´ıticos 294



   1 1 1 1 P0 = 1, , , P1 = , 1, 2 2 2 2 Al evaluar ambos puntos en la funci´on distancia, obtenemos r r 3 1 1 d (x, y, z) = 1 + + = 4 4 2 Consideremos ahora y = x , a partir de las ecuaciones (4) y (5) se obtiene 7 = 0 4 2x + z − 2 = 0

2x2 + z −

Resolviendo el sistema se obtienen los puntos cr´ıticos √ √ √ ! 2 1 2 2 1 P3 = ± , ± ,1 ∓ , 2 4 2 4 2 Al evaluar todos estos puntos en la funci´on distancia, obtenemos √ √ √ ! √ 2 1 2 2 1p 1 ± , ± ,1 ∓ = d 9∓2 2 2 4 2 4 2 2

Como la curva de intersecci´on del paraboloide y el plano es cerrada, las distancias m´axima y minima absoluta del la curva al origen son: dm´ax

√ 1p 9 + 2 2, = 2

dm´ın =

r

3 2

Ejemplo 3 Encuentre el volumen m´aximo de una caja rectangular x2 y 2 z 2 inscrita en el elipsoide 2 + 2 + 2 = 1 con sus caras paralelas a los a b c planos coordenados. Soluci´ on: Sea P(x,x,z) el v´ertice de la caja que esta en el primer octante donde x > 0, y > 0, z > 0. Por la simetria del problema se desea maximizar la funci´on volumen f (x, y, z) = 8xyz sujeta a la condici´on g(x, y, z) = x2 y 2 z 2 + 2 + 2 −1=0 a2 b c Formemos la funci´on auxiliar de Lagrange  2  x y2 z2 F (x, y, z, λ) = 8xyz + λ + 2 + 2 −1 a2 b c 295

y consideremos entonces el sistema. 2λ x = 0 (1) a2 2λ Fy (x, y, z, λ) = 8xz + 2 y = 0 (2) b 2λ Fz (x, y, z, λ) = 8xy + 2 z = 0 (3) c 2 2 y z2 x Fλ (x, y, z, λ) = 2 + 2 + 2 − 1 = 0 (4) a b c Fx (x, y, z, λ) = 8yz +

Multiplicando las ecuaciones (1), (2)y (3) por x, y, x respectivamente, produce 2λ 2 x = 0 (1,1) a2 2λ 8xyz + 2 y 2 = 0 (2,2) b 2λ 8xyz + 2 z 2 = 0 (3,3) c

8xyz +

Entonces , obtenemos 2λ 2 2λ 2 2λ 2 x = 2 y = 2 z = −8xyz a2 b c Para obtener el volumen m´aximo se requiere que x, y, x 6= 0 y λ 6= 0

Concluimos entonces que

y2 z2 x2 = = a2 b2 c2 Sustituyendo esta expresi´on en la ecuaci´on (4), obtenemos 3

x2 a √ − 1 = 0 =⇒ x = a2 3

As´ı, sucesivamente se tiene un u ´nico punto cr´ıtico   a b c P0 = √ , √ , √ 3 3 3 Por lo tanto , la caja tiene un volumen m´aximo dado por   a b 8 c fm´ax √ , √ , √ = √ abc 3 3 3 3 3 296

3.7.3.

Aplicaci´ on al campo de la econom´ıa

Supongase que la producci´on de cierto producto de una compa˜ n´ıa manufacturera es una cantidad Q , donde Q es una funci´on de f (K, L) donde K es la cantidad de capital (o inversi´on) y L es la cantidad de trabajo realizado. Si el precio del trabajo es p, el precio del capital es q y la compa˜ nia no puede gastar m´as de B d´olares, ¿c´omo podemos hallar la cantidad de capital y de trabajo que maximice la producci´on Q? Soluci´ on: Se esperar´ıa que si se incrementa la cantidad de capital o de trabajo, entonces la producci´on deber´a incrementarse; esto es: ∂Q ∂Q ≥0 y ≥0 ∂K ∂L Tambi´en se esperaria que conforme se a˜ nada trabajo a una cantidad dada de capital, obtendremos menos productos adicionales por nuestro esfuerzo, esto es: ∂ 2Q 0 y fxx −2 −2 2 2 2   2 2   1 1 1 1 1 cuyo valor es f = . , , hay un m´ınimo local de f en P5 2 2 2 2 8 Finalmente, estudiemos la funci´on en la frontera del cuadrado: Para x ∈ [0, 1]; y = 0 ; f (x, 0) = 0 =⇒ m´ınimo en este segmento abierto . Ahora, en los v´ertices, f (0; 0) = 0 =⇒ m´ınimo local y f (1, 0) = 0. ′ Para y ∈ [0, 1]; x = 1; f (1; y) = −y 3 =⇒ f (y) = −3y 2 = 0 ⇐⇒ ′′ y = 0; f (y) = −6y; f ′′ (0) = 0 no existen valores extremos en el segmento abierto . Ahora, en los v´ertices f (1, 0) = 0; f (1, 1) = −1. Por lo tanto hay m´ınimos locales en (1, 0),(1, 1). Para x ∈ [0, 1]; y = 1; f (x; 1) = −x3 =⇒ f ′ (x) = −3x2 = 0 ⇐⇒ x = 0; f ′′ (x) = −6x; f ′′ (0) = 0 entonces no existe ni m´aximo ni m´ınimo en el segmento abierto. En los v´ertices x = 0; x = 1 tenemos f (0, 1) = 0; f (1, 1) = −1 =⇒ m´ınimos locales. Para x = 0; y ∈ [0, 1]; f (0; y) = 0 =⇒ m´ınimo local en el segmento abierto.

314